Космос и астрономия

Ответить в тред Ответить в тред
Check this out!
Тред тупых вопросов №148 Zeldovich Edition Аноним 28/03/21 Вск 15:46:10 6442531
zeldovichyab.jpg 50Кб, 280x400
280x400
Спрашиваем то, за что в других местах выдают путёвку в биореактор. Здесь анонимные учёные мирового уровня критически рассмотрят любые гениальные идеи и нарисованные в Paint схемы.

Предыдущий тут: >>640069 (OP)
https://2ch.hk/spc/res/640069.html

Q: Можно быстрее?
A: Можно упасть в пузырь Алькубьерре, NASA уже почти надула его.

Q: Я начитался охуительных историй про уфологию, че делать, нам жопа?
A: Да, тебе жопа, можешь сгонять в зогач или куда оттуда пошлют.

Q: Что будет с человеком в вакууме без скафандра / если он упадет на черную дыру / попробует ступить на поверхность газового гиганта/солнца?
A: Он умрёт.

Q: Почему бы не привязать ракету к воздушному шару или стартовать с горы?
A: Космос - это не как высоко, а как быстро, большая часть энергии ракеты уходит на разгон вбок.
Подробнее тут https://what-if.xkcd.com/58/ (английский) https://chtoes.li/orbital-speed/ (перевод)
Аноним 28/03/21 Вск 16:17:33 6442582
Вот Энцелад срёт гейзерами в вакуум, а они создают какую-нибудь тягу эти гейзеры, влияют на его орбиту?
Аноним 28/03/21 Вск 16:19:03 6442593
А вокруг белых карликов как с радиацией? Есть какие-нибудь вредные лучи?
Аноним 28/03/21 Вск 16:26:32 6442604
>>644258
>Вот Энцелад срёт гейзерами в вакуум, а они создают какую-нибудь тягу эти гейзеры, влияют на его орбиту?

А ты прикинь массу Энцелада и массу выброса.
Аноним 28/03/21 Вск 16:28:31 6442625
>>644259
>вредные лучи?
Лол, как будто есть полезные лучи. Белые карлики светят в рентгене.
Аноним 28/03/21 Вск 16:28:52 6442636
>>644253 (OP)
>пузырь Алькубьерре
Альбукерке же.
Аноним 28/03/21 Вск 16:31:09 6442647
>>644262
> будто есть полезные лучи
Ну, да, вот я порнуху смотрю - фотоны приятные летят в глаза.
Аноним 28/03/21 Вск 16:36:05 6442668
Есть ли какие-нибудь военные преимущества от освоения космического пространства за пределами околоземной орбиты? Или хотя бы на Луне?

От базы на Марсе сильнее не станешь, не? Нахуя на это проёбывать ресурсы?
Аноним 28/03/21 Вск 17:02:43 6442689
>>644266
> военные преимущества от освоения космического пространства за пределами околоземной орбиты?

Минусы:
Чем дальше от Земли аппарат с пушками или ракетами, тем дольше им лететь до Земли, тем дольше времени они находятся в зоне сенсоров, которые могут из зачем и подготовить перехват.
Чем дальше от Земли аппарат с оптикой, тем шакальнее изображение он получает, тем больше нужна камера.

Плюсы:
Чем дальше хуйня-аппарат от Земли, тем сложнее его обнаружить и сбить с Земли. Но дело в том, что нихуя полезного на большом расстоянии ты не сделаешь, будет в безопасности бесполезности. Можно там где-нибудь далеко и высоко командный центр супер-компьютер держать, который будет управлять ядерными ракетами и роботами на Земле. Такое было в романе Филлипа Дика "Вторая модель", где американское правительство и суперкомпьютеры были спрятаны где-то в бункерах на Луне, а оттуда управляли подземными роботами крикунами, которые производились также на беспилотных подземных заводах и вели автономную войну с СССР.
Аноним 28/03/21 Вск 19:09:15 64428710
Реально ли сейчас изменить орбиту астероида размером с челябинский и с вменяемым КВО сделать из него метеорит?
Аноним 28/03/21 Вск 19:48:59 64429611
>>644287
Смотря что понимать под реально - очень гипотетическая ситуация. Если бы мы обнаружили челябинский метеорит за 1 год, за 5 лет, за 10 лет и могли предсказать с точностью, что он упадет в челябинской области - что бы мы сделали? Что бы сделала Россия, что бы сделали другие страны?
Чувак, это очень интересный вопрос. Эвакуировали бы мы область, или же повелись бы на авось сгорит в атмосфере, все по домам ховайтесь? Или же РФ из бюджета выделила бы деньги на перехват астероида? этот вопрос не так прост как тебе кажется.
Аноним 28/03/21 Вск 20:26:25 64429912
>>644253 (OP)
Ебанный стыд...
Во-первых, Альбукерке.
Во вторых, иди нахуй, хули ты испортил шапку?
В третьих это может показаться мелочью, подумаешь, слово поменял, изменил текст максимум на десятимиллионную часть, кажется что все под контролем, но так легко дойти и до эфира в шапке и до поиска нацистских баз в Антарктиде!
Аноним 28/03/21 Вск 21:33:12 64432413
>>644299
Да вообще последние перекаты так себе, еще и ученые повторяются.
Аноним 28/03/21 Вск 23:50:27 64435514
>>644296
Я немного о другом спрашивал. Можно ли из астероида сделать метеорит, с точностью до района города?
Аноним 29/03/21 Пнд 00:04:05 64435715
16169616248090.png 1128Кб, 1500x1000
1500x1000
16169631571160.png 590Кб, 710x710
710x710
Аноним 29/03/21 Пнд 00:10:22 64435816
Аноним 29/03/21 Пнд 00:11:19 64435917
>>644358
Это нейтронка летит в нашу систему. К 25 году земля станет стерильна.
Аноним 29/03/21 Пнд 00:15:05 64436018
>>644359
но судя по фото она уже прилетела, не?
Аноним 29/03/21 Пнд 03:18:21 64436719
>>644266
>военные преимущества от освоения космического
/0
Аноним 29/03/21 Пнд 03:19:00 64436820
>>644287
>Реально ли сейчас изменить орбиту астероида
Нет.
Аноним 29/03/21 Пнд 03:20:34 64436921
>>644324
>еще и ученые повторяются.
А чем тебе не нравится Зельдович? Самый вумный советский ученый эвер, к тому же самоучка.
Аноним 29/03/21 Пнд 03:21:27 64437022
>>644355
>метеорит, с точностью до района города?
Так ты из тех ебанутых, что мечтают кидать астероиды по городам? Съеб в парашу, бактерия.
Аноним 29/03/21 Пнд 04:33:37 64438023
>>644369
Он уже был в одном из прошлых тредов, да и к тому же картинка в шапке всего одна, так неинтересно. За Альбукерке тоже в упор стреляю.
Аноним 29/03/21 Пнд 04:44:54 64438224
161444442111131[...].mp4 1045Кб, 480x480, 00:00:12
480x480
>>644380
>картинка в шапке всего одна, так неинтересно
Аноним 29/03/21 Пнд 07:05:28 64438525
Аноним 29/03/21 Пнд 12:11:15 64442126
>>644370
Можно железные астероиды по ближе к заводу доставлять.
Я мечтаю метеорит только на один город приземлить
Аноним 29/03/21 Пнд 12:25:42 64442327
>>644416 →
Дебил, замедление света прекрасно измеряется. Замедление света в какой-нибудь воде или еще где легко просчитывается, а в вакууме свет не может замедляться в принципе. Мы ведем речь именно про вакуум.
Аноним 29/03/21 Пнд 12:29:27 64442528
>>644423
Какова скорость фотона в железе?
Аноним 29/03/21 Пнд 14:06:07 64444029
>>644425
Ты имеешь в виду просто кусок железа?
Скорость света в веществе зависит от электронной плотности (не путать с плотностью свободных электронов/дырок, которые носители заряда!). Железо д-элемент, а это значит у него много неэкранизированных от ядра электронов и следовательно очень высокая электронная плотность(попутно это обеспечивает ферримагнетизм), от чего будет значительное замедление света больше 10!
Однако железо в чистом виде металл, а это значит электроны проводимости будут частично блокировать прохождение волнового фронта и фотон отразиться, причем как на входе, так и на выходе из кристалла.
В добавок железо не состоит из одного монокристалла и в нем есть примеси и дефекты, которые весьма здорово могут влиять на оптические свойства. Поэтому прямо замедление света в куске железа измерить очень сложно из-за сторонний эффектов.
Аноним 29/03/21 Пнд 17:13:18 64446130
Аноним 29/03/21 Пнд 17:15:40 64446231
>>644440
Фотоны могут пролетать сквозь железо, если у них частота высокая. Так какая скорость у фотона в железе?
Аноним 29/03/21 Пнд 18:40:37 64447732
>>644462
Отражение фотонов зависит от параметров носителей заряда и геометрии кристалла. Так что тут наоборот, что выше частота, чем хуже пролетают внутрь. (Это без учета неконкретного рассеивания.)
По моим прикидкам должно быть около 0,1-0,08 от скорости света. Могу ошибаться.
Аноним 29/03/21 Пнд 23:27:20 64452133
Аноним 30/03/21 Втр 03:07:52 64453734
>>644477
>Так что тут наоборот, что выше частота, чем хуже пролетают внутрь.
А не наоборот ли? Гамма-квант свободно пролетит через 1 мм стали, а ультрафиолет не пролетает.
Аноним 30/03/21 Втр 09:20:12 64455535
>>644537
Хотя вот этот вот не дебил просто дурачок
Аноним 30/03/21 Втр 09:45:09 64455736
>>644253 (OP)
Поясните супердауну, вот допустим есть объект с большой массой, вокруг себя он искривляет или как я понял уплотняет пространство-время. Но у меня вопрос, во что именно уплотняется пространство? Само в себя?
Аноним 30/03/21 Втр 10:49:06 64456437
>>644557
Да. А еще время замедляется.
Аноним 30/03/21 Втр 12:43:11 64458238
Аноним 30/03/21 Втр 14:20:33 64461339
2abcf927994d.jpg 25Кб, 600x348
600x348
Кстати, а Rotary Rocket вообще могла сработать в принципе? Хуй с ним со взлётом, разгонять лопасти очевидно менее эффективно чем сам аппарат. Но само замедление после входа, и посадка на авторотации, хули бы и нет?
Аноним 30/03/21 Втр 19:47:00 64489440
Если я буду жить на орбите Плутона, я проживу в два раза дольше землян?
Аноним 31/03/21 Срд 00:39:56 64501841
>>644894
Ты проживёшь секунд 20.
Аноним 31/03/21 Срд 03:30:23 64502842
Правда, что у СССР были спутники с ядерными реакторами, которые ломались и падали на Землю устраивая миничернобыли?
Аноним 31/03/21 Срд 03:47:25 64502943
>>645028
да, канаду бомбили ещё
Аноним 31/03/21 Срд 05:47:01 64503944
>>644253 (OP)
Существуют ли какие-нибудь теоретические мысли по поводу искусственного создания магнитного поля на планетах.
Даже если они там в духе Лю Цысиня предполагают бурение скважин до центра планеты/спутника и взрывания там 100500 термоядерных зарядов или создание гигантского искусственного спутника.
Аноним 31/03/21 Срд 06:18:43 64504045
>>645039
да, в первых номерных марсотредах обсуждали возможность ебануть соленоид по экватору, даже что-то вроде считали
про исскуственный спутник прохладно, так как поток частиц от солнца создаёт слишком большую тягу, чтобы её компенсировать
бонбы в ядре вообще шиза ебаная
Аноним 31/03/21 Срд 06:23:08 64504146
>>645040
>что-то вроде считали
И сколько амер насчитали?
Аноним 31/03/21 Срд 06:23:25 64504247
Аноним 31/03/21 Срд 06:39:02 64504448
>>645041
не помню что там за изначальные условия были, помню в итоге получилась электростанция на пол тераватта и то ли сотни миллионов, то ли миллиардов тонн медяхи на провод
Аноним 31/03/21 Срд 07:04:57 64504649
>>645044
А ссылки на тред нет?
31/03/21 Срд 07:05:03 64504750
>>645039
Таких теорий дохуя. Но ты даже не попытался искать их.
Аноним 31/03/21 Срд 07:10:59 64504851
>>645047
Расскажи одну, которая тебе больше нравится.
мимо-ск
Аноним 31/03/21 Срд 07:51:47 64505152
>>644462
Гамма-изучение и рентгеновское в веществе распространяется всегда со скоростью превышающей скорость света.
Аноним 31/03/21 Срд 08:14:18 64505453
>>645046
искать не буду, челик
Аноним 31/03/21 Срд 09:18:26 64505954
>>645054
Да и фиг с ним тогда, челик.
Аноним 31/03/21 Срд 09:19:57 64506055
>>645051
>Гамма-изучение и рентгеновское в веществе распространяется всегда со скоростью превышающей скорость света.
Зря ты тут это написал, сейчас наши местные академики сдетонируют.
Аноним 31/03/21 Срд 10:01:53 64506256
>>645051
Типа эффект черенкова? Но к чему ты это?
Аноним 31/03/21 Срд 10:26:25 64506557
>>645062
>Типа эффект черенкова?
Неа, черенковское излучение возникает в ПРОЗРАЧНОМ веществе при движении в нем ЗАРЯЖЕННОЙ частицы. А мы говорим о фотонах, а не о заряженных частицах и железо тоже не прозрачно.
Аноним 31/03/21 Срд 14:46:57 64510458
>>644264
Говорят на Проционе В регулярно порнуху крутят.
Аноним 31/03/21 Срд 23:41:02 64516259
Аноним 31/03/21 Срд 23:48:53 64516360
>>645162
Тянки-проционянки
Аноним 01/04/21 Чтв 06:50:51 64518761
>>645163
как они после сброса оболочки звезды? им там норм?
Аноним 01/04/21 Чтв 16:45:51 64524862
Какой календарь будет на Марсе? Будут ли колонисты отмечать 1 апреля?
Аноним 01/04/21 Чтв 16:56:14 64525163
>>645248
скорее всего примерно как на земле, но на 22 месяца
отмечать будут по земному
Аноним 01/04/21 Чтв 17:06:09 64525464
Можно ли превратить минусы Марса в его плюсы? Как-нибудь позитивно использовать солнечную радиацию и прочее?
Аноним 01/04/21 Чтв 17:13:49 64525665
>>645254
на земле есть места с большим естественным фоном чем на марсе
низкое довление - меньше перенос тепла, проще отапливать
Аноним 01/04/21 Чтв 17:26:44 64526066
>>645256
>на земле есть места с большим естественным фоном чем на марсе

А сколько в цифрах? Сколько зв в час на Марсе в среднем и во время солнечных вспышек и что за места на Земле с таким же кол-вом зв в час?
Аноним 01/04/21 Чтв 17:49:32 64526467
>>645260
на марсе на два порядка выше чем в среднем по земле, картинку с падалкой без пеки не дам

места на земле - это пляжи с монацитовым песочком, непроветриваемые подвалы с радоно проницаемым полом, погреба

с тем учетом что на глубине в десяток-другой метров на всех космических камнях фон одинаковый, а селиться планируют в лавовую трубку - разницы в этом плане почти никакой

про вспышки не скажу, но они емнип прогнозированные
Аноним 02/04/21 Птн 00:50:41 64530568
PIA17601-Compar[...].png 238Кб, 900x1017
900x1017
>>645260
вот тебе картинка с падалкой, не забыл
Аноним 02/04/21 Птн 02:25:06 64530969
>>645264
Ты гуманитарий? Тебя о цифрах спросили.
Аноним 02/04/21 Птн 02:37:20 64531070
>>645309
цифры гуглятся, если это сложно то анонимные академики бессильны
Аноним 02/04/21 Птн 03:19:15 64531271
>>645310
Так ты не знаешь количественные характеристики явлений которые ты берешься сравнивать? Точно гуманитарий.
Аноним 02/04/21 Птн 03:37:47 64531972
Как узнать массу как-то космического тела, если мы знаем его размер, но не знаем плотность?
Аноним 02/04/21 Птн 04:13:06 64532373
>>645319
> Как узнать массу как-то космического тела, если мы знаем его размер, но не знаем плотность?
На глазок прикинуть
Аноним 02/04/21 Птн 04:48:28 64532674
>>645319
если мы знаем размер то знаем и спектр исходя из которого уже можно классифицировать примерную плотность
Аноним 02/04/21 Птн 05:09:01 64532775
>>645323
>На глазок прикинуть
Можешь на глазок прикинуть массу тела с радиусом 1 км и неизвестной плотностью?
Аноним 02/04/21 Птн 05:13:14 64532876
>>645327
1.14 кг на литр, дальше сам
Аноним 02/04/21 Птн 05:33:11 64533277
>>645327
Если там есть второе и более тел, изи.
Аноним 02/04/21 Птн 07:17:20 64533478
загруженное (30[...].jfif 9Кб, 300x168
300x168
Аноним 02/04/21 Птн 21:57:25 64538779
>>644423
>Дебил, замедление света прекрасно измеряется. Замедление света в какой-нибудь воде или еще где легко просчитывается, а в вакууме свет не может замедляться в принципе. Мы ведем речь именно про вакуум.
Ты не понял, а обзываться начал. Что если сама скорость света уменьшается, вот о чем речь. Ты разницы не заметишь, для тебя будет казаться что дальние объекты удаляются, хотя на деле просто свету надо больше времени чтобы их достичь.
Аноним 02/04/21 Птн 22:02:53 64538980
image.png 45Кб, 300x168
300x168
>>645327
Сталь будет тяжелее перьев, потому что сталь тяжелее.
Аноним 02/04/21 Птн 22:15:00 64539081
aloneintheunive[...].jpg 39Кб, 750x422
750x422
Не то чтобы прямо чисто про космос, но про то что инопланетян нет.

Прочитал книжку про поиск инопланетных цивилизаций (альманах разных авторов) и там есть глава, где профессор биологии доказывает что мы с гигантской вероятностью можем быть единственными. Доводы показались логичными.

1. Мы смотрим на все с позиции антропоцентризма и раз на Земле сформировались мы, то на других планетах жизнь тоже должна в конечном итоге породить разум. Но она не должна, у эволюции нет цели, разум вообще не обязан появляться, это продукт адаптации, как полоски у зебры.

2.Поскольку на Земле жизнь появилась почти сразу как для нее возникли условия мы можем предположить что она всегда должна так появляться, но это опять же "судим по себе", это может быть совпадением и аргумент за это - что жизнь у нас возникла только один раз, все существа имеют общего предка. Можно предположить что все другие зародившиеся жизни съедались, но у них было 3.8 миллиарда лет на попытки, и этого не случилось, ни единого примера другой жизни, другого типа.

3. Митохондрии - чтобы стать эукариотами (и получить возможность иметь размер больше микроскопического) должно было произойти пиздецово редкое событие - случайный симбиоз археи и эубактерии. На это потребовались миллиарды лет, неисчислимое количество одноклеточных, больше чем звезд в известной вселенной и их взаимодействий - а это произошло только один раз.

4. Появление фотосинтеза - то же самое, что пункт три, причем понятно, что вероятность того, что произойдут оба события исчезающе мала. Эубактерия научившаяся добывать энергию из света должна была попасть внутрь уже эукариота. Мало того что должно произойти это совпадение (какой на это шанс?) - так еще нужно сделать поправку на то, что чтобы это произошло жизнь должна была пройти точно такой же путь - с митохондриями, совпадениями и так далее. А если митохондрии у них не появились, а только фотосинтез?

4. Многоклеточность - после появления эукариотов прошел миллиард лет пока жизнь стала многоклеточной - опять же, она не обязана была ей становиться, ей и так нормально. Не было стимулов для эволюции - она так и оставалась одноклеточной, 3 миллиарда лет.

Это уже компиляция адовых случайностей, а надо еще не забыть про условия - про тип звезды который подходит, наличие воды, большого спутника, Юпитера, который забирает на себя астероиды - и тем не менее все равно случались вымирания когда более-менее развитая жизнь могла закончится, откатившись до одноклеточных.

5. Люди сами по себе не обязаны были появляться, они стали продуктом резкого изменения условий в которых жили приматы. Сформировавшиеся люди были не обязаны основать цивилизацию - они так и могли шляться и заниматься собирательством. Чуть не вымерли опять же, было бутылочное горлышко, когда численность сократилась до 10к особей, почти вымерли.

Все это можно помножить на будущие риски цивилизации - самоуничтожение в войнах, эпидемиях, деградации от кучи причин и вообще шанса на то что мы "пересечемся" в одном времени с разумной расой, если они вообще с такими раскладами существуют.

-------------------------------------------------------------------------------

Я кратко изложил его тезисы, звучат очень убедительно и печально. Разубеди меня, анон! Скажи где профессор мог налажать, что это хуйня и другие разумные расы должны быть.
Аноним 02/04/21 Птн 23:30:22 64539682
1617395421803.jpg 14Кб, 333x499
333x499
>>645327
> Можешь на глазок прикинуть массу тела с радиусом 1 км и неизвестной плотностью?
Аноним 03/04/21 Суб 08:24:19 64541283
Test
Аноним 03/04/21 Суб 09:37:40 64541484
>>645390
>другие разумные расы должны быть
Учитывая размеры вселенной они скорее всего действительно есть, но слишком далеко от нас что бы это на что-то влияло.
Аноним 03/04/21 Суб 12:17:00 64542285
>>645390
>Я кратко изложил его тезисы, звучат очень убедительно и печально. Разубеди меня, анон! Скажи где профессор мог налажать, что это хуйня и другие разумные расы должны быть.
Let's give it a shot, shall we?
>1. Мы смотрим на все с позиции антропоцентризма и раз на Земле сформировались мы, то на других планетах жизнь тоже должна в конечном итоге породить разум. Но она не должна, у эволюции нет цели, разум вообще не обязан появляться, это продукт адаптации, как полоски у зебры.
Не обязан. По крайней мере не в том виде как у нас. Но почему бы и нет, эволюционно как раз будет иметь преимущество тот кто имеет разум и сможет обхитрить другие виды жизни у себя. И в той или иной степени на нашей планете помимо приматов много видов имеют зачатки разума, вороны хитрые и находчивые, например, могут инструменты использовать. Дай им еще сотню миллионов лет, кто знает?
>2.Поскольку на Земле жизнь появилась почти сразу как для нее возникли условия мы можем предположить что она всегда должна так появляться, но это опять же "судим по себе", это может быть совпадением и аргумент за это - что жизнь у нас возникла только один раз, все существа имеют общего предка. Можно предположить что все другие зародившиеся жизни съедались, но у них было 3.8 миллиарда лет на попытки, и этого не случилось, ни единого примера другой жизни, другого типа.
"Маловероятность" контрится количеством планет. Почти у каждой звезды есть планетная система, как оказалось, 100 миллионов звезд на среднюю галактику, 100 миллиардов галактик во вселенной. Разделим на 10 000, будем считать что только одна из 10000 звезд имеет планету в зоне златовласки. 1014 планет с возможностью зарождения жизни. 100 триллионов планет как минимум, у которых миллиарды лет на это.
>3. Митохондрии - чтобы стать эукариотами (и получить возможность иметь размер больше микроскопического) должно было произойти пиздецово редкое событие - случайный симбиоз археи и эубактерии. На это потребовались миллиарды лет, неисчислимое количество одноклеточных, больше чем звезд в известной вселенной и их взаимодействий - а это произошло только один раз.
Миллиарды лет.
>4. Появление фотосинтеза - то же самое, что пункт три, причем понятно, что вероятность того, что произойдут оба события исчезающе мала. Эубактерия научившаяся добывать энергию из света должна была попасть внутрь уже эукариота. Мало того что должно произойти это совпадение (какой на это шанс?) - так еще нужно сделать поправку на то, что чтобы это произошло жизнь должна была пройти точно такой же путь - с митохондриями, совпадениями и так далее. А если митохондрии у них не появились, а только фотосинтез?
Опять шансы и опять - миллиарды лет.
>4. Многоклеточность - после появления эукариотов прошел миллиард лет пока жизнь стала многоклеточной - опять же, она не обязана была ей становиться, ей и так нормально. Не было стимулов для эволюции - она так и оставалась одноклеточной, 3 миллиарда лет.
Вот ты сам к этому и подошел.
>Это уже компиляция адовых случайностей, а надо еще не забыть про условия - про тип звезды который подходит, наличие воды, большого спутника, Юпитера, который забирает на себя астероиды - и тем не менее все равно случались вымирания когда более-менее развитая жизнь могла закончится, откатившись до одноклеточных.
Выше написано.
>5. Люди сами по себе не обязаны были появляться, они стали продуктом резкого изменения условий в которых жили приматы. Сформировавшиеся люди были не обязаны основать цивилизацию - они так и могли шляться и заниматься собирательством. Чуть не вымерли опять же, было бутылочное горлышко, когда численность сократилась до 10к особей, почти вымерли.
Похуй на людей, выше написано. Аргумент от антропоцентризма, разумным видом могли стать те же вороны.
>Все это можно помножить на будущие риски цивилизации - самоуничтожение в войнах, эпидемиях, деградации от кучи причин и вообще шанса на то что мы "пересечемся" в одном времени с разумной расой, если они вообще с такими раскладами существуют.
Похуй на самоуничтожение и прочее. У Вселенной есть непреодолимая грань называемая скоростью света, далеко летать просто крайне долго. Даже если наша галактика битком забита разумными существами, они могут просто не долететь до нас пока мы 50 лет фонили радиоволнами.
Аноним 03/04/21 Суб 12:21:40 64542386
Аноним 03/04/21 Суб 12:39:27 64542487
>>645414
Такой вариант даже печальнее.

>>645422
Вот с воронами динозаврами тоже сложно. Это даже выглядит контрпримером - да, птицы научились использовать простые орудия, как элемент адаптации, но это ни к чему не привело, не появилось разумных птиц, за все миллионы лет (там много проблем - яйца, полет накладывает ограничения на размер мозга, нет такой социальности, как у людей).

Еще есть птицетазовые динозавры - про них часто пишут что они очень умные, социальные и если бы не вымирание то тогда бы...

А по факту они существовали миллионы лет, значительно дольше людей с их предками - а даже до уровня шимпанзе в своей энцефализации не дотянули, то есть рост мозгов это прям такая четкая адаптация на конкретные изменившиеся условия, которые опять же ни с каким другим видом, кроме нашего, не приключились.

>У Вселенной есть непреодолимая грань называемая скоростью света, далеко летать просто крайне долго. Даже если наша галактика битком забита разумными существами, они могут просто не долететь до нас пока мы 50 лет фонили радиоволнами.

Все так



Аноним 03/04/21 Суб 12:58:56 64542688
Почему Роскосмос не может запустить Союз в облет Луны? Корабль есть, РН "Протон" есть. Можно ж и пилотируемые, и беспилотные полеты осуществлять, это ж готовая победа на первом этапе новой лунной гонки.
Почему рынок суборбитальных запусков, судя по всему, нинужен? А если нужен, почему тогда на нем все так вяло?
Аноним 03/04/21 Суб 14:58:18 64543089
>>645426
Какой в этом смысл?
Аноним 03/04/21 Суб 15:14:06 64543490
>>645430
туристов отвозить, показать что можем, можно кубсаты выбросить на пути, в окошко посмотреть, кратеры посчитать, первые исторические русские за пределами орбиты земли и облёт вокруг Луны россией.
Аноним 03/04/21 Суб 15:21:24 64543691
>>645434
>туристов отвозить
Туристов? А какая себестоимость у такой турпутевки?
Аноним 03/04/21 Суб 16:08:00 64544092
>>645436
Запуск Союза (то есть, и корабль, и одноименная ракета, и обслуживание/инфраструктура) к МКС -- 70 млн долларов. Запуск РН "Союз" -- 40 млн, "Протон" -- 70. То есть, около 100 млн долларов ориентировочно. И то, это уже не себестоимость идет, так что по факту даже меньше должно быть.
Аноним 03/04/21 Суб 16:17:06 64544193
>>645440
А если опустить вопрос цены, пусть это очередная давайпососательная скрепа будет. Еды в нем хватит на неделю? От радиации защитит? Будет чем маневрировать?
Аноним 03/04/21 Суб 16:22:37 64544294
>>645441
диды на дремучем говне летали, а свеженькое судно нисможет?
Аноним 03/04/21 Суб 16:25:44 64544395
>>645440
>Запуск Союза (то есть, и корабль
На Союзе к Луне не слетать увы не выйдет.
Аноним 03/04/21 Суб 16:26:26 64544496
>>645442
>диды на дремучем говне летали
К Луне и обратно? Советские деды никогда не летали к Луне.
Аноним 03/04/21 Суб 16:30:38 64544697
>>645444
мурриканские диды, при чем здесь совки
на радиацию тогда хуй забили, просто летели по прогнозу, еды на три рыла тоже весят хуйню
Аноним 03/04/21 Суб 16:44:05 64544898
>>645441
>>645443
Так Союз же разрабатывался для полетов к Луне. Человеков планировали отправить вокруг Луны на Союзе 7К-Л1, но не успели до того, как это сделали США, а потом стало неинтересно.
Аноним 03/04/21 Суб 16:52:39 64544999
>>645441
Касательно радиации: на Луне около 1,4 мЗв в день, на МКС -- 0.5-0.7 мЗв в день. И ничего, летают и живут по 100-300 суток. Зонд-5 облетел Луну и был в космосе 7 суток от запуска до спуска. Примерно как пара недель на МКС по дозе, получается
Аноним 03/04/21 Суб 17:28:15 645450100
Буран.jpg 151Кб, 1280x637
1280x637
Спейсач, нужен вердикт на тему пикрелейтед - это возможно, или хуйня? Бэкграунд - это из сериала For All Mankind, про альтернативную историю где Королёв дожил до 1983 года, а Алексей Леонов был первым человеком на Луне. И там есть космодром для "Бурана" на Сахалине в 1983 году. В диалоге двух персонажей говорится что "Буран" это копия "Шаттла", причем твердотопливные ускорители один в один. Такой вариант реален?
Аноним 03/04/21 Суб 17:52:54 645452101
>>645450
Буран это и была ухудшенная копия шаттла, почему нереально-то? Кстати, а хули на картинке тогда нарисованы 4 боковых ускорителя и центральный бак с движками, как на Энергии?
Аноним 03/04/21 Суб 17:53:34 645453102
>>645444
Черепахи летали зато.
Аноним 03/04/21 Суб 21:06:43 645467103
>>645390
Позволю себе немношк критики

Intro:
>профессор биологии доказывает что мы с гигантской вероятностью можем быть единственными. Доводы показались логичными.
Да, но без практики это всё 0
1. Верно
2. Верно, но не до конца
3. Скипну ибо "да, пиздецово редкое, но случилось"
4. Скипну по причине п.3
5. Бля а чет еще будет кроме того что было в 3-4 пункте?

>звучат очень убедительно и печально
Не спеши впадать в грусть и тоску мил человек, лучше эпикурейцев изучи

>Скажи где профессор мог налажать
Детерминизм в вероятностном подходе
>это хуйня и другие разумные расы должны быть
Всё предельно просто. Вероятность возникновения нас 1 шанс из 2,04*10390

>Откуда взялась такая вероятность? Ну мы посчитали опираясь на всё то, что есть сегодня. А что будет послезавтра?

Ирония тут вот в чем. Прямо сейчас на какой-нибудь суперземле хер его знает где могут тусить бактериальные маты. Или строиться строматолиты. Мы их никогда не найдем, упустим и т.д. и т.п. IF они там были, но мы проебались, влияет ли это на реальность? На модели наши да, но не на реальность. Именно поэтому нам нужен космос. Нужно влезть во все дырки, устроить ФГДС, Ректороманоскопию и спектрографию каждому шарику до какого не можем дотянуться. А до какого шарика дотянуться не можем, нужно дотянуться и проверить его.

Ты (и я) умрем наверняка раньше, чем найдется что-то интересное. Но вероятность того что оно найдется онон в данной точке времени может поменяться. Мы с тобой об этом не узнаем. Однако, стоит ли опираться на одну лишь вероятность?
Аноним 03/04/21 Суб 21:42:34 645471104
это натурфелософов тред?
Аноним 03/04/21 Суб 22:45:17 645479105
>>645448
7К-Л1 это не Союз. И уж тем более не современный Союз, который в отличие от первых Союзов 7К-ОК для длительных автономных полетов не пригоден, максимальная длительность автономного полета всего 3 суток.
А для лунного полета требуется модификация теплозащиты, модификация систем связи и модификация систем терморегулирования. Нет, в принципе физика не запрещает, но никак не в рамках "возмем стоимость полета к МКС и докинем разницу до Протона".
Аноним 03/04/21 Суб 22:47:21 645480106
>>645479
Он же ещё вроде бы только облёт может, без выхода на орбиту, или я ошибаюсь?
Аноним 03/04/21 Суб 22:58:33 645483107
>>645479
В плане не Союз?
И что случится при более длительном полете? Электроника откажет? (не должна вроде) Питания не хватит? (аналогично) Кислорода? (думаю, решить можно) Что именно мешает? Насчет необходимости модификации вопросов нет, но что конкретно в плане автономности ограничивает?
И да, думаю, при серийной постройке стоимость разработки модификации окажется сравнительно небольшой. Полагаю, единственное, что сильно увеличит стоимость (и массу) отдельно взятого корабля -- это теплозащита. А так в 100-150 миллионов за полет уложиться можно, если лететь не один раз
Аноним 03/04/21 Суб 23:05:36 645484108
>>645480
Да, только облет. Для выхода на орбиту даже тупо по дельте потребуется многопуск, причем в формате минимум Протон + аналог Зенита по грузоподъемности.
Что-то на уровне связки Союз+Фрегат/Бриз+ДМ-3
И разумеется, потребуется дорабатывать разгонники под возможность стыковки. Плюс опять же, Фрегат или Бриз придется модифицировать под длительный автономный полет.
Аноним 03/04/21 Суб 23:36:14 645485109
>>645483
>Насчет необходимости модификации вопросов нет, но что конкретно в плане автономности ограничивает?
Как минимум ресурс системы жизнеобеспечения. Даже для Союз-Аполлона ставили дополнительные регенераторы атмосферы.
http://www.gctc.ru/main.php?id=301
>при серийной постройке стоимость разработки модификации окажется сравнительно небольшой.
При очень сильно серийной. Во первых там не только теплозащита. Терморегуляция космических аппаратов это большая проблема, а тепловой режим на трассе Земля-Луна и на низкой орбите отличаются очень сильно. Связь на больших расстояниях это проблема. Плюс нужно как минимум один аппарат в беспилотном варианте запустить, а его себестоимость по первым заказчикам размазать.
И при этом никакой очереди туристов нет. Собственно у нас сравнительно недавно озвучивали, что если очередь будет, то такое можно и устроить. Но в мире не так много людей, готовых заплатить ~100 лямов долларов за облет Луны.
Ну и да, я не хочу разочаровывать, но Протоны уже все, производство свернуто и в планах только отстрелять уже изготовленные ракеты, которые все уже давно расписаны. То есть или нужно заново разворачивать производство, или пускать на Ангаре, которая гораздо дороже.
Аноним 04/04/21 Вск 00:03:20 645487110
>>645485
Много протонов осталось?
Аноним 04/04/21 Вск 01:17:43 645488111
>>645319
Способов узнать массу - миллион и тележка, размер и плотность для большинства не нужны. Да хоть банально по траектории этого тела и соседних. Обычно определяют по возможности большим количеством разных методов, чтобы надёжней было.
Аноним 04/04/21 Вск 01:33:46 645489112
Gravitatsionnoe[...].jpg 128Кб, 813x610
813x610
Блядь.
Мне 33 года и я тупой.
Как сука работает гравитация?
Какого хуя все планеты не засосало в Солнце?
Какого хуя Луна не ебнулась на Землю.
Поясните за гравитацию.
Аноним 04/04/21 Вск 02:26:57 645491113
>>645326
>если мы знаем размер то знаем и спектр
Какой спектр?
Аноним 04/04/21 Вск 02:27:49 645492114
>>645488
>Да хоть банально по траектории этого тела и соседних.
Как по "траектории и средним" узнать массу тела?
Аноним 04/04/21 Вск 02:54:23 645493115
>>645491
смотря чем измеряли размер
Аноним 04/04/21 Вск 06:50:04 645505116
>>645489
>Какого хуя все планеты не засосало в Солнце?
>Какого хуя Луна не ебнулась на Землю.
Потому что промахиваются
Аноним 04/04/21 Вск 08:23:11 645510117
>>645487
13 штук, но все распланированы под конкретные нагрузки, лишних нет.
Аноним 04/04/21 Вск 09:36:10 645512118
>>645485
Вот тоже хотел задать вопрос: а на кой Протоны свернули, если Ангара гораздо дороже? Из-за токсичного топлива?
Аноним 04/04/21 Вск 09:55:52 645513119
>>645512
>а на кой Протоны свернули, если Ангара гораздо дороже?
Производственная цепочка Протонов разорвана, РФ не может их сама производить.
Аноним 04/04/21 Вск 10:31:55 645514120
Аноним 04/04/21 Вск 11:14:38 645515121
>>645512
Из-за топлива, из-за желания использовать имеющуюся в Плесецке инфраструктуру.
>>645513
Не пизди. Решение о переходе на Ангару было принято задолго до прекращения производства Протона.
Аноним 04/04/21 Вск 11:31:05 645516122
>>645515
А что экономически выгоднее - выводить Протоном или Ангарой?
Аноним 04/04/21 Вск 12:50:34 645527123
>>645510
А потом всё? Или будет чем ещё запускать?
e 04/04/21 Вск 13:42:25 645529124
eee
Аноним 04/04/21 Вск 14:44:15 645533125
>>645516
Ангара в три раза дороже Протона. Может, со временем изменится, как знать
>>645527
Ангарой той же
Аноним 04/04/21 Вск 15:15:40 645535126
>>645449
>в день, в день.
В час
Аноним 04/04/21 Вск 15:20:35 645536127
>>645488
> Да хоть банально по траектории этого тела и соседних.
Летит некое тело в межзвездной среде, без спутников. Плотность его неизвестна, известен только размер. Как узнать массу?
Аноним 04/04/21 Вск 15:24:20 645537128
>>645536
>Летит некое тело в межзвездной среде, без спутников. Плотность его неизвестна, известен только размер. Как узнать массу?
Что это за тело такое и если это не звезда то как мы его вообще заметили?
Аноним 04/04/21 Вск 15:31:51 645539129
>>645387
>Что если сама скорость света уменьшается
С хуя ли она должна уменьшаться? Просто потому, что это пришло в голову какому-то ноунейм-дауну на дваче? Иди штаны поменяй и таблетки прими, ебалай. Скорость света у него, блядь, уменьшается.
Аноним 04/04/21 Вск 15:34:12 645541130
>>645489
Гравитация ничего не сосет, лол. Это искривление пространства-времени массой объектов.
Аноним 04/04/21 Вск 15:43:46 645542131
>>645537
>Что это за тело такое

А хуй его знает, летит себе и летит. Так как узнать его массу?
Аноним 04/04/21 Вск 15:49:46 645543132
>>645542
Если это звезда - то можно примерно так почувствовать, если нет то хуй знает, да и видно это вряд ли будет.
Аноним 04/04/21 Вск 15:54:21 645544133
12824952062.jpg 786Кб, 1600x1200
1600x1200
центробежная гр[...].png 48Кб, 3532x2176
3532x2176
>>644253 (OP)
Центробежная гравитация
Анон, зачем делать дорогостоящие кольца или цилиндры, когда можно ограничиться космическим коромыслом?
Аноним 04/04/21 Вск 15:55:57 645545134
>>645543
>если нет то хуй знает

А какие вообще есть способы узнать массу тел?
Аноним 04/04/21 Вск 15:56:57 645546135
h31LQ.png 6Кб, 480x400
480x400
Вот ещё похожее нашел
>>645544
Аноним 04/04/21 Вск 15:57:34 645547136
>>645545
Если не известна плотность и оно не обращается вокруг другого тела то никаких. Можно наверно попробовать гравитационные волны половить, но у нас нет таких точных детекторов.
Аноним 04/04/21 Вск 15:57:55 645548137
>>645544
>Анон, зачем делать дорогостоящие кольца или цилиндры, когда можно ограничиться космическим коромыслом?

Никто не делает ни кольца, ни цилиндры, ни коромысла. Человеку нечего делать в космосе, а роботам искусственная сила тяжести не нужна.
Аноним 04/04/21 Вск 15:58:06 645549138
>>645547
В кольцо больше людей влезет
Аноним 04/04/21 Вск 15:58:51 645550139
Аноним 04/04/21 Вск 16:00:11 645551140
>>645493
>смотря чем измеряли размер
Радиолокацией например.
Аноним 04/04/21 Вск 16:01:57 645552141
>>645549
Ну да вопрос цели. А если цель это исследовательское судно на 5 человек например? Коромысло хорошо тем что оно может быть очень длинным, то есть там сила кориолиса будет меньше выражена и к тому же его не надо быстро крутить. Ну и дешево же.
Аноним 04/04/21 Вск 16:13:17 645557142
CuFkGWsYK8A.jpg[...].jpg 150Кб, 1000x983
1000x983
Аноним 04/04/21 Вск 16:14:09 645558143
>>645552
>А если цель это исследовательское судно на 5 человек например?
Покажи пальцем где предлагали строить колесо на 5 человек.
Аноним 04/04/21 Вск 16:15:54 645559144
Аноним 04/04/21 Вск 16:25:08 645563145
1c89a74570eee50[...].jpg 94Кб, 960x739
960x739
И еще старенькая американская идея >>645558
Аноним 04/04/21 Вск 17:13:31 645568146
>>645535
А не много? Откуда инфа?
Аноним 04/04/21 Вск 18:11:07 645573147
>>645551
ну вот по отраженному радиоспектру можно состав поверхности, и далее классифицировать и экстраполировать в плотность и массу
Аноним 04/04/21 Вск 20:26:56 645585148
Можно ли урезать учёным их паёк до минимума? Разве там не должны работать сугубо идейные люди?
Аноним 04/04/21 Вск 20:37:47 645586149
>>645585
Я сейчас тебя до минимума урежу ежжи
Аноним 04/04/21 Вск 20:41:43 645587150
Аноны, пожалуйста, подскажите литературы. Есть что нибудь, про математические модели в астрофизике почитать? Недавно купил машину для вычислений, вот хочу космосы помоделировать, а мат.часть не знаю. Выручайте.
Аноним 04/04/21 Вск 21:14:56 645589151
>>645585
А смысл? Если один Уэбб стоит как вся зарплата всех физиков мира за пару лет?
Аноним 04/04/21 Вск 21:18:15 645590152
>>645390
Я бы вообще ещё один вопрос добавил, насколько вообще нормальна наша научно техническая революция.
Аноним 04/04/21 Вск 22:27:45 645593153
Аноним 05/04/21 Пнд 02:56:27 645602154
>>645573
>по отраженному радиоспектру можно состав поверхности,
Расскажи, как это сделать и почему сейчас никто так не делает?
Аноним 05/04/21 Пнд 02:57:03 645603155
1544680695241.jpg 36Кб, 415x731
415x731
>>645587
>Аноны, пожалуйста, подскажите литературы
Аноним 05/04/21 Пнд 10:57:49 645622156
>>645593
В прямом. У нас НТР результат дикого стечения обстоятельств в локальном регионе.
Китай же 3000 лет предел и дрочил стабильность.
Возможно каждая сотая звезда имеет планету с жизнью, каждая десяти тысячная планету с разумом. Но технологическая цивилизация одна на каждый миллиард лет жизни каждой галактики.
Короче Великие Древние это мы и есть.
Аноним 05/04/21 Пнд 11:58:45 645625157
>>645390
Можно ещё добавить что подходящие для жизни планеты могут быть только у третьего поколения звезд, и далеко не все подходящие звёзды находятся в галактической зоне обитаемости.
Аноним 05/04/21 Пнд 14:33:11 645644158
>>645539
Откуда я знаю с чего она уменьшается. Но ты может быть перестанешь изрыгать желчь и задумаешься о таком аргументе? Потому что если ты будешь дальше быть токсичным ублюдком, разговаривать с тобой не буду.
Аноним 05/04/21 Пнд 14:35:16 645646159
>>645557
>>645563
>ни солярок, ни радиаторов
Видимо, дальше "рассказали знакомому художнику" дело у проектов не зашло.
Аноним 05/04/21 Пнд 14:39:23 645647160
>>645467
Спасибо.

В целом согласен, но "пиздецово редкое но случилось" - речь же как раз про то что не стоит ориентироваться на это и исходить из того что раз случилось, то точно случится еще раз - спокойно может не случится.

То есть получается что просто какая-то одноклеточная жизнь в принципе должна где-то быть, а что-то сложное и тем более разумное - может и в принципе не появиться вообще нигде, или появляться чудовищно редко и далеко. Вариант "одни в нашей галактике" - вполне реален.

>>645625

>подходящие для жизни планеты могут быть только у третьего поколения звезд

А кстати почему? Не первый раз слышу. Про первое поколение понятно, но почему не может быть у второго?
Аноним 05/04/21 Пнд 14:45:31 645649161
>>645647
У второго поколения всё еще не достаточно металличности
Аноним 05/04/21 Пнд 14:47:08 645650162
>>645646
Она в атмосфере просто вращается, вон вихри на краях видишь. Без радиаторов неплохо остывает.
Аноним 05/04/21 Пнд 15:19:45 645651163
Почему у Луны почти нет атмосферы? У Титана есть, больше чем у Земли, у Плутона (!) есть хоть кое-что, что помешало Луне?
Аноним 05/04/21 Пнд 15:35:26 645653164
>>645622
>У нас НТР результат дикого стечения обстоятельств в локальном регионе.
>Китай же 3000 лет предел и дрочил стабильность.

Все просто. Китайская цивилизация, как и все первые, развилась на плодородных почвах бассейнов Хуанхэ и Янцзы. Обе несут огрмное количество ила, особенно Хуанхе и разливаясь они делают почву чрезвычайно плодородной. Китайские царства благодаря им первыми в мире решили проблему голода. Нет голода - стабильность.
Аноним 05/04/21 Пнд 15:37:33 645654165
>>645647
>То есть получается что просто какая-то одноклеточная жизнь в принципе должна где-то быть, а что-то сложное и тем более разумное - может и в принципе не появиться вообще нигде

Вселенная бесконечна, а значит бесконечно число жизней в ней.
Аноним 05/04/21 Пнд 15:38:23 645655166
>>645651
>Почему у Луны почти нет атмосферы?
Почему нет? Есть у Луны атмосфера.
Аноним 05/04/21 Пнд 16:19:48 645657167
819px-Solarsyst[...].png 270Кб, 819x1024
819x1024
>>645651
>Почему у Луны почти нет атмосферы
Она улетела.
Аноним 05/04/21 Пнд 17:32:10 645658168
>>645651
На титане холодно
Аноним 05/04/21 Пнд 17:55:52 645659169
Посоветуйте, где можно найти подробный разбор Птолемеевской и Коперниканской системы, со всеми входными данными, эпициклами и вытекающим матаном. Заебало видеть эти идеализированные картинки, где у Коперника ровные кружки вокруг Солнца, и это-де орбиты планет, а не их деференты
Аноним 06/04/21 Втр 02:01:19 645701170
>>645654
Во-первых мы не знаем, бесконечна ли она, а во вторых - если есть ближайшая вторая разумная жизнь в галактике Андромеды - нам от этого вообще ни жарко ни холодно, мы не встретимся с ней никогда.
Аноним 06/04/21 Втр 02:19:47 645702171
>>645644
Твой аргумент - говно собачье, как и содержимое твоей гнилой черепушки, хуепутало ебаное. Ссу на тебя и на твой говноаргумент.
>Потому что если ты будешь дальше быть токсичным ублюдком, разговаривать с тобой не буду.
Лол, ты заплачь еще, педовка блядь.
Аноним 06/04/21 Втр 02:25:58 645703172
>>645654
Пиздабол, в современной космологии Вселенную рассматривают как трехмерную гиперсферу, т.е. как конечную по объему, но не имеющую краев.
Аноним 06/04/21 Втр 03:29:32 645704173
>>645701
>Во-первых мы не знаем, бесконечна ли она
Верно. Это лишь гипотеза.
>если есть ближайшая вторая разумная жизнь в галактике Андромеды - нам от этого вообще ни жарко ни холодно, мы не встретимся с ней никогда.
Тоже верно.
Аноним 06/04/21 Втр 03:30:54 645705174
>>645703
>Пиздабол, в современной космологии Вселенную рассматривают как трехмерную гиперсферу, т.е. как конечную по объему

И сколько пк3 у нее объем?
Аноним 06/04/21 Втр 06:23:17 645710175
>>645702
Раз не можешь по делу писать. Репорт. Такому скоту как ты тут не место.
Аноним 06/04/21 Втр 07:09:41 645711176
>>645710
Он токсичен, просто игнорируй таких.
Аноним 06/04/21 Втр 08:30:37 645716177
>>645710
Кек, ебать ты униженка обоссаная. Мочератору уже сделал минет?
>>645705
>И сколько пк3 у нее объем?
Это пока неизвестно.
Аноним 06/04/21 Втр 08:57:45 645719178
>>644253 (OP)
Здравствуйте, звёздные люди. Обесните мне, пожалуйста, не очень умному, на пальцах, почему нельзя просто без задней мысли взять и полететь на орбиту, как на машине? Обязательно какие-то ракеты, какие-то ступени. В чём главные проблемы?
Аноним 06/04/21 Втр 09:24:59 645725179
>>645719
>В чём главные проблемы?
В Ньютоне. Он праклятый придумал свой второй закон и теперь мы страдаем.
Аноним 06/04/21 Втр 09:29:35 645727180
>>645725
> второй закон
Он везде работает или только поблизости от Земли? Иначе говоря, чтобы мне на аналогичную разницу скоростей разогнаться в открытом космосе, мне тоже придётся столько же топлива сжигать, как при выходе на орбиту?
Аноним 06/04/21 Втр 09:39:04 645730181
>>645719
>>645720 →
> Дороги нет, ехать не по чему.
А если проложить монорельсу на магн. левитац. от уровня моря до вершин гор?
>>644253 (OP)
> Космос - это не как высоко, а как быстро, большая часть энергии ракеты уходит на разгон вбок.
Таким образом на разгон вбок мы будем тратить энергию атомной электростанции. Эта монорельса у нас будет вместо первой ступени, на ней будем разгонять космический аппарат со второй ступенью, который в конце разгона, на вершине горы, будет врубать уже реактивный движок и разгоняться дальше уже в стратосфере.

Хватит ли денег налогоплательщиков?
Аноним 06/04/21 Втр 09:48:05 645732182
>>645727
>Он везде работает
Он работает со всеми макроскопическими телами (все что массивней частицы) движущимися с неоче большими (меньше сотых долей скорости света). Он применим ко всем телам в Солнечной системе.
>чтобы мне на аналогичную разницу скоростей разогнаться в открытом космосе, мне тоже придётся столько же топлива сжигать, как при выходе на орбиту?
Если на ракету не действуют никакие другие силы тяги ракетного двигателя то затратив одинаковое количество топлива ракета наберет большую скорость, чем такая же ракета в гравитационных полях.
Аноним 06/04/21 Втр 09:52:47 645733183
>>645732
> чем такая же ракета в гравитационных полях
Воот. А этот... мне про второй закон Ньютона затирает. Главные препятствия - гравитация и трение об атмосферу же. Трение можно минимизировать обтекаемым корпусом, а с гравитацией никак. Только энергию вкачивать. Поэтому нужны разгонные монорельсы из поста выше. Реактивная тяга для первой ступени - тупиковый путь. ИМХО. Пруф ми вронг.
Аноним 06/04/21 Втр 09:58:37 645734184
>>645733
> А этот... мне про второй закон Ньютона затирает
Естественно. Ведь ты же спросил
>>645719
>Здравствуйте, звёздные люди. Обесните мне, пожалуйста, не очень умному, на пальцах, почему нельзя просто без задней мысли взять и полететь на орбиту, как на машине?
Если стартуешь с Земли то без него никак не обойтись.
Аноним 06/04/21 Втр 09:59:26 645735185
>>645733
> разгонные монорельсы
Гиперлуп?
Аноним 06/04/21 Втр 10:01:35 645736186
>>645735
Дораха. Обтекаемый корпус дешевле. Налогоплательшики охуеют оплачивать герметичную трубу до стратосферы.
Аноним 06/04/21 Втр 10:10:35 645738187
>>645736
А что если сделать такой трамплин в герметичной трубе в которой создан вакуум, на него поставить тележку на магнитном подвесе и разогнавшись на трамплине пробить выходную пленку и лететь в космос на тележке?
Аноним 06/04/21 Втр 10:14:13 645739188
>>645738
> пробить выходную пленку
Не надо. Достаточно уравновесить давление в трубе с давлением на выходе (вершина горы). Но сама труба и откачка - это нерационально дорого.
Аноним 06/04/21 Втр 10:15:43 645740189
Ваши мнения по этому поводу?

6 апр - РИА Новости. Российская частная компания "КосмоКурс", планировавшая создать многоразовый суборбитальный комплекс для космического туризма и построить космодром в Нижегородской области, будет закрыта, сообщил РИА Новости генеральный директор компании Павел Пушкин.

"Компания ликвидируется, работникам объявлено о сокращении", - рассказал он.

Причиной закрытия стали непреодолимые трудности с согласованием требований к проекту космодрома с местными властями и невозможность получения от Минобороны необходимой нормативной документации для проектирования суборбитальной туристической ракеты. О трудностях ранее РИА Новости рассказывал Пушкин.

В настоящее время в "КосмоКурсе" работает 50 конструкторов ракетно-космической техники. Об увольнении их предупредили за два месяца до сокращения, после чего начнется процедура ликвидации организации.

Как сообщили РИА Новости в "Роскосмосе", руководством госкорпорации принято решение сохранить коллектив конструкторов, для чего ведутся поиски путей решения проблемы.

"Руководство "Роскосмоса" в курсе той ситуации, которая сложилась вокруг компании "КосмоКурс". Сейчас ведутся консультации с одним из научных учреждений, входящих в периметр "Роскосмоса", о включении в его состав уникального коллектива", - сказали в пресс-службе госкорпорации.
Непосредственно решением проблемы занимается генконструктор России по средствам выведения, замгендиректора головного научного института "Роскосмоса" ЦНИИмаш Александр Медведев.

Компания "КосмоКурс" была создана в 2014 году для разработки ракетно-космической техники и предоставления услуг туристических суборбитальных полетов. Для проведения пусков планировалось создание космодрома под Нижним Новгородом.

В свое время компания была резидентом фонда "Сколково", получила лицензию "Роскосмоса" на космическую деятельность.

Проект "КосмоКурса" предполагал разработку одноступенчатой возвращаемой ракеты и семиместного космического корабля. Первый полет планировался на 2025 год. Туристы должны были провести в невесомости около пяти минут.

Билет в космос ориентировочно оценивался в 200-250 тысяч долларов. Компания планировала проводить до 115 пусков в год, отправляя в путешествие до 700 человек. Ожидалось, что в основном это будут иностранцы.

Ранее по разным причинам были закрыты или снизили свою активность до минимума такие компании, как спутникостроительная компания Dauria, разработчик легких ракет "Лин Индастриал", группа компаний "Галактика", многократные смены руководства и планов преследуют владельца комплекса "Морской старт" - космическое предприятие группы S7 - S7 Space.
Аноним 06/04/21 Втр 10:20:05 645741190
Аноним 06/04/21 Втр 10:23:05 645743191
>>645740
Они бы еще в уганде развивали науку и ракетостроение. Рашка - это северная африка, местные диктаторы просто не позволят что-либо делать.
Аноним 06/04/21 Втр 10:46:55 645746192
В перигелии скорость у Паркера 192 км/с. Наблюдаются ли какие-то релятивистские эффекты при движении на такой скорости?
Аноним 06/04/21 Втр 11:08:48 645753193
>>645730
>>645733
https://en.wikipedia.org/wiki/Space_fountain
https://en.wikipedia.org/wiki/Skyhook_(structure)
https://en.wikipedia.org/wiki/Launch_loop
И в целом https://en.wikipedia.org/wiki/Non-rocket_spacelaunch

>Хватит ли денег налогоплательщиков?
Да запросто. Только никто их выделять на это не будет. Зачем вкладываться в космос если можно те же деньги тратить на войну и яхты?
Аноним 06/04/21 Втр 11:10:00 645754194
Аноним 06/04/21 Втр 11:10:53 645755195
>>645746
Они наблюдаются даже у обычных спутников на орбите земли.
Аноним 06/04/21 Втр 11:13:28 645756196
>>645746
>В перигелии скорость у Паркера 192 км/с. Наблюдаются ли какие-то релятивистские эффекты при движении на такой скорости?
Безусловно.
На такой скорости Лоренц-фактор составляет 1.00032. Уже вполне заместно, кстати, для точных приборов.
Я не знаю какую роль там ОТО играет из-за близости к Солнцу, посчитайте кто-нибудь за меня.
Аноним 06/04/21 Втр 11:14:45 645758197
>>645756
>Я не знаю какую роль там ОТО играет
Вытягивает зонд в колбаску.
Аноним 06/04/21 Втр 11:21:19 645763198
Аноним 06/04/21 Втр 11:30:15 645768199
>>645756
>1.00032
Ты 3 нуля после запятой проебал
Аноним 06/04/21 Втр 11:53:07 645776200
image.png 148Кб, 1073x270
1073x270
>>645768
Разве? Я в км/с считал.
Аноним 06/04/21 Втр 11:55:12 645777201
>>645763
Вакуумные трубы это большой геморрой, мы даже гиперзалупу реалистично не можем построить пока что, так что ни фонтан, ни трамвай, ни петля пока не светят. Самое реалистичное - сверхтяжами скайхук вывести, наверное.
Аноним 06/04/21 Втр 12:19:19 645781202
>>645452
Я тебе больше скажу: у шаттла центральная часть была тупо подвесным баком, а тут под ним видны движки. То есть, они даже тут обосрались. Даже чертеж не смогли нормально переделать.
>>645450
> И там есть космодром для "Бурана" на Сахалине в 1983 году.
Это уже 1991 год, тащемта.
> В диалоге двух персонажей говорится что "Буран" это копия "Шаттла", причем твердотопливные ускорители один в один. Такой вариант реален?
Нет, потому что нахуй оно надо.
Аноним 06/04/21 Втр 12:39:02 645790203
Если космический зонд летит из Солнечной системы к другой звезде, в направлении совпадающем с орбитального движения Солнца, то скорость Солнца добавится к собственной скорости зонда, а если зонд летит в противоположенном направлении?
Аноним 06/04/21 Втр 13:12:12 645793204
>>645790
Упадет в центр галактики
Аноним 06/04/21 Втр 13:35:01 645795205
Аноним 06/04/21 Втр 13:35:13 645796206
загруженное (1).jfif 5Кб, 225x225
225x225
Аноним 06/04/21 Втр 16:10:16 645808207
be.jpeg.jpg 171Кб, 776x448
776x448
Сап, экспертач.
Может быть такое, что ячеистая структура вселенной вызвана тем, что темная энергия обладает отрицательной гравитацией и расталкивает материю? По идее примерно почувствовал что при такой картине обычная материя должна собираться в тонкие стенки вокруг пузырей из темной энергии, чем бы это ни было, а вся вселенная должна расширяться.
Кто-нибудь всерьез это уже анализировал или мне бежать за нобелевкой?
Аноним 06/04/21 Втр 16:19:56 645810208
>>645808
>темная энергия
А что это?
Аноним 06/04/21 Втр 16:22:59 645811209
>>645808
Ячеистая структура объясняется гравитацией. Расталкивание для "стенок" и "нитей" не нужно.
Аноним 06/04/21 Втр 17:00:12 645817210
>>645808
> расталкивает материю
ну ты или волен с терминологией или нихуя не понял о чем говоришь
Аноним 06/04/21 Втр 17:48:01 645824211
>>645450>>645452
>В диалоге двух персонажей говорится что "Буран" это копия "Шаттла", причем твердотопливные ускорители один в один. Такой вариант реален?

Буран нихуя не копия Шаттла, а его концептуальное подобие на уровне планера. Идея в том, что космоплан запускается только по необходимости, чтобы спутники с орбиты пиздить или огурцов возить. РН Энергия можно запустить и без бурана (на все 100 тонн полезной нагрузки), а с Шаттлом ты всегда запускаешь ебалу размером со сверхтяж, имея 24т полезной нагрузки. Это плюс.

Минус тоже есть: маршевые двигатели у бурана проёбываются, у шаттла не проёбываются, т.е. шаттл в теории дешевле. Но с реюзом шаттла всё получилось крайне геморройно, а проекты реюза двигателей Энергии тоже были, так что тут всё неоднозначно.
Аноним 06/04/21 Втр 17:56:28 645825212
>>645824
>Но с реюзом шаттла всё получилось крайне геморройно, а проекты реюза двигателей Энергии тоже были, так что тут всё неоднозначно.
Всегда забавляла эта маневренность бураноблядей - бумажные проекты развития бурана-энергии у них несравненно лучше реальности шаттла и все получилось бы именно как планировалось
Аноним 06/04/21 Втр 18:36:25 645831213
>>645450
>причем твердотопливные ускорители один в один
У Энергии-бурана не было твердотопливных ускорителей. Были керосин-кислородные.

>>645825
Если тебе дохуя приебалось посрать на Россиюшку и поклониться западному барину, то там всё то же самое:
- Shuttle-C был попыткой выкинуть из шаттла космоплан, отказаться от реюза маршевых двигателей и запускать на боку стотонные хуёвины, как это делали совки с Скиф-ДМ.
- Нынешняя SLS по сути является полным аналогом Энергии и станет им окончательно, когда в Block2 запилят керосиновые бустеры вместо твердотопливных.
- Реюз маршевых двигателей пилят на юловской Vulcan. Перекочует ли эта схема на пока ещё салфеточную SLS Block2 - пока неизвестно, но концепция такая есть.

Ну а с какого раза нормально начинает полноценно работать бумажный проект - видно по истории F9, которая на первых запусках была перетяжеленным говном без реюза и сменила кучу версий. Так что два запуска не показатель, а политота к космическим системам отношения не имеет. Может завтра Байден подохнет, Камала объявит социалистический полпотизм, и штаты накроются пиздой вместе со всей их космической отраслью - это не значит, что отрасль плохая.
Аноним 06/04/21 Втр 19:01:00 645838214
>>645824
>Буран нихуя не копия Шаттла, а его концептуальное подобие на уровне планера.
Настолько оконцептуаленное, что начали со 100% копии и потом итеративно подгоняли под то, что советская промышленность реально могла сделать — отсюда и появление четвертого двигателя, и вынос их с челнока на центральный бак, и увеличение числа боковых ускорителей с 2 до 4. Генеральный конструктор программы, кстати, никогда не скрывал, что Буран скопирован.

>Копирование, как это указано в предыдущем ответе, было, безусловно, совершенно сознательным и обоснованным в процессе тех конструкторских разработок, которые проводились и в процессе которых было внесено, как уже было указано выше, много изменений и в конфигурацию, и в конструкцию. Основным политическим требованием было обеспечение габаритов отсека полезного груза, одинакового с отсеком полезного груза "Шаттла".
http://www.buran.ru/htm/archivl.htm

>Идея в том, что космоплан запускается только по необходимости, чтобы спутники с орбиты пиздить или огурцов возить. РН Энергия можно запустить и без бурана (на все 100 тонн полезной нагрузки), а с Шаттлом ты всегда запускаешь ебалу размером со сверхтяж, имея 24т полезной нагрузки. Это плюс.

Это плюс для Энергии, но охуенный минус для самого Бурана, т.к. он по факту был просто сверхдорогим многоразовым обтекателем весом под 70 тонн. Задачи, под которые он якобы был сделан, существовали только в голове у партийных маразматиков, никакой "по необходимости" просто не было.

Вся программа — апофеоз советского карго-культа, когда сумасшедшие диды из ЦК приказали сделать самолётик как у американцев, и конструкторы бодро взяли под козырёк и побежали закапывать миллиарды рублей в землю на абсолютно бессмысленном проекте.
Аноним 06/04/21 Втр 19:22:04 645839215
Что-то не могу найти инфу. Какова сейчас масса оригинального международного прототипа килограмма?
Аноним 06/04/21 Втр 19:22:46 645840216
>>645838
>т.к. он по факту был просто сверхдорогим многоразовым обтекателем весом под 70 тонн
Врешь, русофоб! Буран - это ещё и многоразовый недоРБ.

>и конструкторы бодро взяли под козырёк и побежали закапывать миллиарды рублей в землю на абсолютно бессмысленном проекте.
Помню, на НК рассматривались конспирологические теории о том, что Глушко с какого-то момента подкупили враги, ибо считай ВСЕ его поздние проекты как будто специально спроектированы под закапывание тонн денег без какой-либо отдачи.
Аноним 06/04/21 Втр 19:51:23 645846217
>>645831
Здоровья тебе и добра, анон.
Аноним 06/04/21 Втр 20:40:17 645851218
>>645838
>Это плюс для Энергии, но охуенный минус для самого Бурана, т.к. он по факту был просто сверхдорогим многоразовым обтекателем весом под 70 тонн.
А он им и должен быть. Десятиместная фура с двадцатитонным багажником - это очень нишевая конструкция, для которой задачи есть только в небольшом проценте случаев. Это важные и нужные задачи (типа отремонтировать телескоп Хаббл или даже вернуть его на землю для техобслуживания), но выпадают они редко. А 95% случаев это извоз огурцов до МКС, извоз тяжелых спутников до ГСО, зонды всякие и прочая рутина. Космоплан-фура в таких задачах нахуй не нужон, он только мешает. Поэтому совки в общем поступили мудро, сделав его доп.опцией. Да, весьма гипертрофированной доп.опцией в угоду каргокультистам из ЦК, но всё-таки основные деньги были въёбаны на ракету-носитель, которая имела ценность сама по себе.
Аноним 06/04/21 Втр 21:56:40 645860219
>>645622
>Великие Древние это мы и есть.

Какое самомнение, однако
Аноним 06/04/21 Втр 22:27:25 645867220
>>645781
>Нет, потому что нахуй оно надо.
Тебе ж сказали, что КГБ тупо спиздило чертежи у американцев и по ним тупо сделали копию, потому что самим придумать никак.
Аноним 06/04/21 Втр 22:47:46 645872221
Пожалуйста, не продолжайте с этим >>645867 общаться, с ним же и так все понятно.
Аноним 07/04/21 Срд 00:21:04 645885222
Что у вас тут? Азаза, опять бураносрач.

>>645838
>Вся программа — апофеоз советского карго-культа, когда сумасшедшие диды из ЦК приказали сделать самолётик как у американцев, и конструкторы бодро взяли под козырёк и побежали закапывать миллиарды рублей
Кукаретическую хуйню сказал. Решение делать самолётик шло не от ЦК, а от конструкторов, в условиях неполной инфы и тотальной паранойи. На пике холодной войны абсолютно всё в первую очередь рассматривалось как оружие. Ключевую роль в решении сыграла инфа про режим AOA (Abort Once Around, когда он летит из Ванденберга и садится через виток - требование впоследствии дропнутое военными, но бесполезные дельта-крылья переделывать уже никто не стал), а также анализы ЦНИИМаш и ИПМ, в которых считали что шаттл имеет чисто военное назначение и будет в основном использоваться для таких одновитковых миссий, т.к. декларируемое гражданское не имеет никакого смысла (так и было, лол). Также часть публично доступной в журналах инфы про шаттл не была принята во внимание, часть анализа была просто ошибочной. В общем в семидесятых попильное назначение такой хуиболы было абсолютно неочевидным.

С дивана предположу, что роль сыграло также желание повторить инфраструктуру. Потому что чёрт его знает что там американцы бы сделали на имеющихся от постройки шаттла оборудовании, производственных цепочках и технологиях, а догонять пришлось бы уже через шаг.

Короче почитай детальный анализ от Хендрикса, чтобы хуйни не пороть.
https://www.thespacereview.com/article/3873/1
https://www.thespacereview.com/article/3876/1
Аноним 07/04/21 Срд 00:24:57 645887223
>>645840
Глушко в Э-Б был исполнитель и не был первопричиной. Решение было сформировано на основе работы ЦНИИМаш и мнения Келдыша, в меньшей степени Оцохимского и Сихарулидзе из ИПМ.
Аноним 07/04/21 Срд 04:47:26 645901224
>>645887
Все советские ракетные технологи родом из Германии, от фон Брауна, Тиля, Греттрупа.
Аноним 07/04/21 Срд 09:40:15 645924225
>>645808
Крупномасштабная структура Вселенной связана с веществом (обычным и темным), с флуктуациями плотности вещества в ранней Вселенной, а не с темной энергией. Темная энергия - это не вещество, она не состоит ни из каких частиц и не собирается ни в какие пузыри, она равномерно распределена по всему пространству и ведет себя как космологическая постоянная.
Аноним 07/04/21 Срд 11:01:30 645935226
>>645777
> Вакуумные трубы
Обычные ракеты летают безо всякой вакуумной трубы, просто у неё форма обтекаемая, что конечно не убирает нагрузку от трения полностью, но уменьшает. Так и с космотрамваем, по монорельсу разгоняется обтекаемой формы аппарат. Благодаря тому, что разгоняется он на электротяге, расходы энергии на трение не так затратны, остаётся только проблема с перегревом. Значит под поверхностью носового обтекателя закладываем трубки с фреоном и на том же электричестве его охлаждает холодильник (тепловой насос), а в хвосте разгоняемого аппарата ставим тележку с баком, в котором собираем разогретую холодильником вспомогательную жидкость. В конце разгона вспомогательная тележка отделяется и по петле монорельсы уезжает обратно на базу. Собственно говоря, можно вообще сделать отделяемую от тележки ступень космического аппарата, а вся тележка многоразово циркулирует по монорельсе, запуская по несколько рейсов в сутки. Знай только заменяй горячий охладитель на холодный в охладительном баке. Соответственно и сложный обтекатель космическому аппарату не нужен. Вверху вторая ступень вылетает из середины разгонной тележки. Обтекатель у самого космического аппарата простой, там уже атмосфера пореже, не успеет нагреться. Скорость там уже охерительная, пара минут и на орбите. Ну разумеется, я щас не точно говорю, может и не прав насчёт разогревов. Исхожу из того, что там наверху есть т.н. термосфера, но сегодняшние ракеты так быстро её пролетают, что не успевают нагреться об неё.

Так что ящитаю, именно космотрамвай самый реальный проект. Реальнее космопращи (скайхука).
Аноним 07/04/21 Срд 11:09:22 645936227
>>645935
Рельсу для трамвая будет держать святой дух?
Аноним 07/04/21 Срд 11:13:09 645937228
>>645936
Почему ты не читаешь хотя бы дискасс сначала? Внимательно пробеги по ссылкам вверх и увидишь, где и на чём лежит монорельс. И почему именно там.
Аноним 07/04/21 Срд 11:14:49 645938229
>>645901
А американские откуда?
Аноним 07/04/21 Срд 11:29:02 645939230
>>645938
От Годдарда, они-то сами все сделали. (Сарказм с целью высмеять)
Аноним 07/04/21 Срд 11:43:08 645941231
Может ли существовать планета с пресным мировым океаном?
Аноним 07/04/21 Срд 11:48:21 645943232
>>645941
Может.
Но вместо золота там будет натрий в цене.
Аноним 07/04/21 Срд 12:11:26 645949233
>>645941
Вряд ли. Вода хороший растворитель и будет вымывать всякое говно в виде солей и ионов. А говна в породах полно.
Хотя может быть планета-океан где все говны осели.
Но в целом, если в протопланетном достаточно кислорода для воды, то и натрии с хлорами будут наверняка.
Аноним 07/04/21 Срд 12:13:24 645951234
>>645943
Ха, отличный ответ.
Аноним 07/04/21 Срд 12:14:04 645952235
Фотон - это частица или волна? Почему он в одних экспериментах ведет себя как волна, а в других как частица?
Аноним 07/04/21 Срд 12:31:59 645956236
Аноним 07/04/21 Срд 12:43:44 645958237
>>645956
К чему ссылки? Расскажи своими словами.
Аноним 07/04/21 Срд 12:46:28 645959238
>>645958
Как тебе такое объяснение - частиц нет, это все волны, просто воспринимаются как частицы.
Аноним 07/04/21 Срд 13:21:33 645966239
>>645959
>частиц нет, это все волны, просто воспринимаются как частицы
Можно с тем же успехом сказать "это всё частицы, просто воспринимаются как волны", весь вопрос в слове "воспринимаются".

На самом деле человеческая интуиция просто говно, она не работает на объектах не из макромира. Фотон не частица и не волна, он ДРУГОЕ.
Аноним 07/04/21 Срд 13:23:54 645969240
16101075192340.jpg 105Кб, 927x487
927x487
Аноним 07/04/21 Срд 13:27:06 645971241
>>645959
>это все волны,
Так есть же эксперимент который доказывает, что фотон это частица. Как с этим быть?
Аноним 07/04/21 Срд 14:25:32 645973242
>>645971
А есть эксперимент, которые доказывает что волна. Что теперь делать будем?
Аноним 07/04/21 Срд 14:27:14 645974243
>>645973
>. Что теперь делать будем?
Хороший вопрос. Может подеремся?
Аноним 07/04/21 Срд 14:44:13 645976244
>>645973
Нет таких экспериментов. Есть только эксперименты, показывающие, что свет изучается и поглощается дискретными порциями.
Аноним 07/04/21 Срд 15:14:38 645986245
laser-interfera[...].jpg 26Кб, 800x280
800x280
LaserInterferen[...].jpg 2785Кб, 1950x1950
1950x1950
>>645976
>Нет таких экспериментов.
Приехали.
Аноним 07/04/21 Срд 15:22:53 645987246
Аноним 07/04/21 Срд 20:32:36 646053247
>>644253 (OP)
Нахуя вы такие умные? Может ли спейсач создать шедевр? Написать симфонию?
Аноним 07/04/21 Срд 20:47:08 646057248
Аноним 07/04/21 Срд 20:48:00 646059249
>>646053
>Нахуя вы такие умные?
Так получилось вследствие изначальных параметров Вселенной после Большого Взрыва. Какой-то причины на это нет.
>Может ли спейсач создать шедевр?
Вероятность такого события есть, разумеется, но пока такого в обозримом окружении не наблюдалось.
>Написать симфонию?
Исключено, это противоречит фундаментальным законам физики.
07/04/21 Срд 23:20:41 646092250
>>645489
потому что тела(планеты) имеют свою скорость и она высока, чтобы из присосала к солнцу она должна быть нулевой или околонулевой, солнце по этой же причине не присосалось к СМЧД в центре милкивея ну и ещё потому что мы на каком то из рукавов вроде летаем тупо кайф летим пару сотен км/с )))
08/04/21 Чтв 00:06:19 646098251
>>645777
Объясните плиз , это трос? Типо подвешиваешь груз и он поднимается на крюке? А что на орбите? Должно же это чем-то компенсироваться?
Аноним 08/04/21 Чтв 08:04:57 646109252
>>646098
Компенсируется подцепляющимися прилетающими хуйнями
Аноним 08/04/21 Чтв 09:15:27 646114253
Аноним 08/04/21 Чтв 09:37:12 646118254
>>645951
Пасиб. >>645949 Твой ответ тоже неплох. Моё увожение!
Аноним 08/04/21 Чтв 10:30:45 646123255
Как изменится климат на Земле если внезапно по какой-нибудь фантастической причине 10-20% атмосферы испарится в космос. Интересует не сама возможность такого а конкретно последствия для климата. Станет теплее? Холоднее. А то что дышать станет труднее, как в горах это понятно.
Аноним 08/04/21 Чтв 11:05:09 646124256
>>646123
>Станет теплее?

Нет. Главный аккумулятор тепла на Земле это океан, он хранит и переносит на порядки больше тепла, чем атмосфера.
Аноним 08/04/21 Чтв 12:59:15 646140257
>>646135
>пока мы не поднимем давление выше 0,1 - 0,2 атм
Ты снова забыл принять свои лекарства?
Аноним 08/04/21 Чтв 13:28:45 646146258
>>646124
Не аккумулятор и не тепла. Это главный фактор климатических/погодных условий.
Аноним 08/04/21 Чтв 13:41:55 646149259
Аноним 08/04/21 Чтв 13:59:12 646150260
Аноним 08/04/21 Чтв 14:13:24 646153261
Аноним 08/04/21 Чтв 17:27:41 646174262
>>645952
Это квантовая частица.
>Почему он в одних экспериментах ведет себя как волна, а в других как частица?
Это из-за квантовой суперпозиции. Она свойственна всем элементарным частицам.
Аноним 09/04/21 Птн 03:23:41 646250263
>>646174
>Это из-за квантовой суперпозиции.
А может все проще? Может дело в том, что мы сожем зарегистрировать фотон только поглотив его веществом детектора, а "пролет" фотона зафиксировать не уничтожив фотон не можем?
Аноним 09/04/21 Птн 05:12:39 646256264
Поясните про СТО. Допустим корабль летит со световой скоростью, ему в лицо светит свет звезды, к которой он летит. Для наблюдателя на корабле, планеты вокруг звезды будут обращаться вокруг этой звезды в два раза быстрее или с нормальной скоростью?
Аноним 09/04/21 Птн 09:20:41 646263265
>>646256
В два раза быстрее.
Аноним 09/04/21 Птн 09:54:44 646266266
>>646256
в охуилиард раз быстрее только за счет лоренц фактора
Аноним 09/04/21 Птн 10:05:01 646268267
>>646256
Нахуй тебе эта информация, ты идиот?
> Для наблюдателя на корабле, планеты вокруг звезды будут обращаться вокруг этой звезды в два раза быстрее или с нормальной скоростью?
Они будут стоять на месте, ведь ты уже разогнался до световой, поздравляю.
Аноним 09/04/21 Птн 10:07:45 646269268
>>646268
Нафиг ты такой токсичный тут высираешься? Вернись на порашу и там говном кидайся, если нормально общаться не умеешь.
Аноним 09/04/21 Птн 10:20:38 646270269
0803d65c45940a8[...].jpg 12Кб, 220x220
220x220
>>644253 (OP)
Как мириться с мыслью, что на экзопланеты - новые уникальные миры, на которых могут быть формы жизни, животные и растения, которых не видывал свет, но на моем веку до них никто не доберется и я так и не смогу уйти дальше фантазий по их поводу?

Если жизнь зарождается в воде, а механизмы отбора +- одинаковы, не значит ли это, что мы увидим схожие пути развития жизни на других планетах? Т.е по идее мы можем увидеть формы жизни, очень похожие на наши земные, потому что на других планетах жизнь так же зарождалась в воде, переходила на сушу и отбиралась такими же природными законами
Аноним 09/04/21 Птн 10:21:44 646271270
>>646270
> Как мириться с мыслью
При помощи широко известных стадий принятия неизбежного.
Аноним 09/04/21 Птн 10:30:01 646272271
>>646270
Становись мильордером, под конец жизни генную терапию в китае на увеличение теломер и омоложение делай и живи хоть тыщу лет, глядишь и увидишь на своем веку новые уникальные миры.
Аноним 09/04/21 Птн 10:35:09 646273272
>>646272
> и живи хоть тыщу лет
Стив Джобс передаёт привет.
С сообщением "на ужин - макароны".
Аноним 09/04/21 Птн 10:37:13 646274273
>>646273
>Стив Джобс передаёт привет.
Я сказал действовать противоположно тому как он вел себя и позволить передовой медицинской науке действовать, а не лечить давно продиагностированный рак народными средствами как он.
Аноним 09/04/21 Птн 10:43:50 646275274
>>646274
Ваще не ибёт, лечил он или нет. Помрёшь всё равно. ПРИМИ это.
Аноним 09/04/21 Птн 10:43:55 646276275
>>646273
Так он год лечил рак травками и медитацией.
Аноним 09/04/21 Птн 10:44:32 646277276
>>646270
Уверуй. Господь с тобой!
Аноним 09/04/21 Птн 11:00:55 646279277
>>646275
Я давно принял, зачем ты мне это пишешь?
Аноним 09/04/21 Птн 11:07:02 646280278
>>646279
Блять, я что, в личку пишу? Ну извини.
Аноним 09/04/21 Птн 11:28:11 646282279
image.png 1333Кб, 771x681
771x681
>>646270
>Т.е. по идее мы можем увидеть формы жизни, очень похожие на наши земные
Похожие разве что с биохимической точки зрения. На земле в прошлом жили хуёвины абсолютно не похожие на современные, на другой планете глобальные вымирания могут идти совсем по-другому и привести к совсем иным формам жизни.
Аноним 09/04/21 Птн 11:32:29 646284280
>>646282
Ну чем сложнее жизненная форма - тем ниже шанс увидеть ее аналогию в другом мире. Я думаю, встретить что-то похожее на земных рыб шансы высокие, рыбы вроде почти всегда были в таком же виде
Аноним 09/04/21 Птн 11:37:10 646286281
>>646282
Антропный принцип гипотетически говорит нам, что разумная жизнь всё равно будет гуманоидной: прямохождение, развитая моторика передних конечностей, большой мозг, вынесенный наверх в голову.
Аноним 09/04/21 Птн 11:43:55 646288282
>>646286
Ну мы на разумную жизнь не залупаемся, про животных пиздим. Там уже вариативность выше. Мне честно говоря слабо верится в существование разумной жизни, как-то слишком невероятно это, хоть и понимаю, что математически шансы нихуевые, учитывая бесконечность вселенной. Что уж там, в одной только нашей галактике экзопланет миллионы или миллиарды
Аноним 09/04/21 Птн 11:45:03 646289283
>>646286
А чем тебя цефалоподы не устраивают? Тоже вариант.
Аноним 09/04/21 Птн 11:48:34 646291284
>>646286
>Антропный принцип гипотетически говорит нам, что разумная жизнь всё равно будет гуманоидной
Это замечательно, но разумная жизнь на земле развилась в определенных условиях, которых на экзопланете может и не быть.
Аноним 09/04/21 Птн 11:53:05 646292285
>>646289
Да я-то и не возражаю. Няшил бы и цефалоподочку, если бы прилетела.
>>646291
Антропный принцип не расписывает детали. Только конечный результат. Они вообще могут быть экзотической жизнью в ледяной матаново-жидкостной среде. Принцип описывает конечный результат. Гипотетически.
>>646288
Оу, тогда умолкаю.
Аноним 09/04/21 Птн 11:54:14 646293286
>>646292
>Няшил бы и цефалоподочку, если бы прилетела.
Если можно доверять японцам, экспертам в этом деле, то скорее всего цефалопод бы няшил тебя.
Аноним 09/04/21 Птн 12:03:21 646296287
>>646292
>Няшил бы и цефалоподочку, если бы прилетела.
Да я бы и собаку разумные грибы... лишь бы хоть что-то прилетело
Аноним 09/04/21 Птн 12:10:26 646297288
>>646296
>Да я бы и собаку разумные грибы...
Если можно доверять японцам, экспертам в этом деле, то скорее всего разумный гриб бы няшил тебя.
Аноним 09/04/21 Птн 12:18:29 646298289
>>646297
Если они мне протеины вычислят, то я не против
Аноним 09/04/21 Птн 12:26:35 646299290
1534693024-1943[...].jpg 99Кб, 1200x483
1200x483
c7792d13eb8057e[...].png 2282Кб, 1920x1080
1920x1080
Возможно ли существование пикрилов ирл?
Аноним 09/04/21 Птн 12:31:33 646300291
>>646299
Огромных боевых человекоподобных роботов? Да, возможно.
Аноним 09/04/21 Птн 12:31:37 646301292
>>646299
ебанинума под них нет
Аноним 09/04/21 Птн 12:43:47 646303293
>>646300
По инфе из вымышленной вселенной трансформеров, они происходят из планеты Кибертрон, которая состоит из металлов и порождает металлические формы жизни

Это сложно представить, гораздо проще прикинуть, что какие-то белковые гуманоиды создали этих роботов

Кстати, по сюжету одной из книг про Трансформеров, они аналогично не представляют, что жизнь может быть из чего-то помимо металла
Аноним 09/04/21 Птн 12:49:57 646304294
>>646303
А, ну это в /c/ лучше обсуждать тогда, тут надо разбираться.
Аноним 09/04/21 Птн 14:11:57 646317295
>>646299
Сейчас невозможно
Аноним 09/04/21 Птн 16:33:50 646340296
Аноним 09/04/21 Птн 18:01:04 646349297
Matrix-4-no-ten[...].jpg 84Кб, 960x540
960x540
>>646299
>IRL
What is real. How do you define real? If you're talking about what you can feel, what you can smell, what you can taste and see, then real is simply electrical signals interpreted by your brain.
Аноним 09/04/21 Птн 21:10:37 646380298
Аноним 10/04/21 Суб 10:08:47 646454299
Анон, я недавно где-то нашёл новость, что обнаружена экзопланета, которая по размеру соответствует Юпитеру, однако анализ атмосферы показал, что вопреки всем возможным предположениям, рассчётам и моделям образования планет, поверхность оказывается твёрдой. Типа, планета железо-каменная как минимум на поверхности.
Сейчас вот вспомнил, а нагуглить больше инфы не могу.
Что ты об этом слышал, анон? Что про это знаешь?
Аноним 10/04/21 Суб 10:15:11 646455300
>>646454
или тебе приснилось, или речь была про хтоническую, с которой сдуло атмосферу, но даже так размер юпа это пиздёж чистрй воды или погрешность порядка на три с половиной
Аноним 10/04/21 Суб 10:28:38 646457301
>>646455
Нет, не хтоническая. Там ещё акцент делался на том, что по всем известным нам моделям такого быть не может, но тем не менее, согласно анализу поверхность твёрдая, а не газовая. Мол, или это сфера Дайсона или не всё мы знаем о планетообразовании. Или что-то ещё.
Я, собственно, и хочу больше новостей по теме найти, ибо может там просто каменные ветра дуют в атмосфере? Или там разбился недавно астероидный пояс прямо на орбите этой планеты, и эти камушки на за атмосферу приняли, например
Аноним 10/04/21 Суб 11:02:28 646465302
>>646454
>обнаружена экзопланета, которая по размеру соответствует Юпитеру, однако анализ атмосферы показал, что вопреки всем возможным предположениям, рассчётам и моделям образования планет, поверхность оказывается твёрдой.

Как же по-твоему такое можно узнать?
Аноним 10/04/21 Суб 11:29:17 646472303
Кто-то читал книжки Владимира Сурдина?
С детства фанатею фантазиями про космос, планеты там всякие, смотрел документалки на эту тему, был фаном Стивена Хокинга. Чем можно заняться в этом направлении, чтобы увлекательно и без дрочева науки типа физики? Чисто обывателю пофантазировать, без тонкостей узнать чего
Аноним 10/04/21 Суб 12:25:26 646487304
>>646472
>без дрочева науки типа физики?
В физике нет ничего сложного.
Аноним 10/04/21 Суб 12:37:43 646488305
>>646487
Нет ничего сложного, разве что если ты остановился на Ньютоне или хотя бы на Максвелле.
Аноним 10/04/21 Суб 12:42:20 646491306
>>646488
> на Ньютоне или хотя бы на Максвелле.
Этого вполне хватит! Ну, еще немного Лоренца и точно хватит.
Аноним 10/04/21 Суб 12:43:44 646493307
>>646472
На самом деле наука кажется сложной просто от того что ты не знаешь как ее применять. Все эти ньютоны и джоули и паскали вполне понятны когда ты видишь их на практике.
Посмотри простой и наглядный научпоп, начиная с Veritasium, Vsauce, Thunderf00t, там разжеванно и понятно, дальше уже сможешь читать по интересам и сможешь понимать FermiLab и PBS SpaceTime.
Аноним 10/04/21 Суб 12:49:18 646495308
>>646487
>>646493
Мне просто лень углубляться. Физику теоретическую я знаю, я говорю о том, чтобы не было всякой ебли и расчетов, просто астрономическая теория
Аноним 10/04/21 Суб 12:51:06 646496309
>>646495
Как вкатишься - расчеты перестанут казаться еблей, базарю.
Но вообще глянь PBS SpaceTime, если уверен в себе. Если просто простенькое то какой-нибудь Astrum.
Аноним 10/04/21 Суб 12:53:47 646498310
Аноним 10/04/21 Суб 12:56:02 646500311
Gg
Аноним 10/04/21 Суб 13:35:57 646506312
>>646465
А как определяют наличие/отсутствие того или иного элемента в атмосфере той или иной далёкой планеты?
Аноним 10/04/21 Суб 14:21:52 646514313
Аноним 10/04/21 Суб 14:22:35 646515314
Аноним 10/04/21 Суб 14:51:04 646516315
>>644299
>до поиска нацистских баз в Антарктиде
Там только космодром, а базы на Луне и Марсе.
Аноним 10/04/21 Суб 15:15:34 646519316
>>646491
А вдруг он хочет освоить не только электромагнитное взаимодействие и гравитационное (справедливости ради, они покрывают 99.999% явлений из обычной жизни), но и сильное и слабое? Как ты "просто" объяснишь, например, почему глюонов 8 типов, а не 6 и не 9, без отсылки ко всяким специальным унитарным группам и матрицам Гелл-Манна? Там уже нихуя не просто становится.
Аноним 10/04/21 Суб 15:17:49 646520317
>>646506
по спектру
ещё раз: каменная плонетка размером с юп это условно пять юпов по массе

если это не хтонический гигант то не набрать атмосферу в процессе формирования эта дичь может только если в протопланетном диске не было газа , что никак невозможно, вот совсем никак
Аноним 10/04/21 Суб 15:19:02 646521318
>>646519
>А вдруг он хочет освоить не только электромагнитное взаимодействие и гравитационное
Там все , от Ниьютона.
Аноним 10/04/21 Суб 15:20:33 646522319
>>646519
>Как ты "просто" объяснишь, например, почему глюонов 8 типов
А это никто не объяснит, в это надо прост верить.
Аноним 10/04/21 Суб 15:47:09 646535320
>>646520
>по спектру
Вот именно. Вот только подобный метод слишком поверхностен. В комбинации с наблюдением объёма и гравитации мы можем сделать предположение - не факт, а предположение - о соответствующей массе тела. Спектр, даже если он 100% верен, показывает лишь спектр самой поверхности планеты, в данном случае, атмосферы планеты. Так что с чего ты взял, что эта планета вся целиком железокаменная?
Аноним 10/04/21 Суб 15:59:35 646536321
>>646535
А какая она по-твоему, надувная, что ли? И да, это спектр самой поверхности, а не атмосферы, если только там атмосфера не как у Венеры.
Аноним 10/04/21 Суб 16:02:33 646537322
>>646535
> Так что с чего ты взял, что эта планета вся целиком железокаменная?
так а в чем новость тогда и в чем вопрос в этом ттв итт

все газовики под слоем лёгких елементов железокаменные, только вот примерно почувствовать это можно через магнито и гравиметрию, через слой водорода на юпах спектр не увидеть

итого опять выходит что либо новость желтушный кал, или ты её так прочитал, тебе веры несколько больше, наверное
Аноним 10/04/21 Суб 18:07:46 646551323
>>646537
>все газовики под слоем лёгких елементов железокаменные
покажи спектральные фото юпитера сатурна - заценим, из чего у них ядра под газовой юбкой
Аноним 10/04/21 Суб 18:53:59 646554324
>>646536
>спектр самой поверхности
по интернету оттуда прислан?
Аноним 10/04/21 Суб 19:35:22 646557325
>>646554
Просто у атмосферы есть такой показатель, как оптическая толщина, примерно соответствующий ее вкладу в общий спектр. У Земли и Марса это пшик, у всяких Меркуриев, Плутонов и Лун вообще ~0, а вот у Венеры самой поверхности и не видно, через телескоп можно только атмосферу понюхать.
Аноним 10/04/21 Суб 19:56:26 646560326
>>646557
вот нафига тупыи американцы с луны камни возили. можно ж состав через телескоп узнавать. да ещё через световые годы. а с на другой земли можно телескопом состав травки изучить.
Аноним 10/04/21 Суб 20:32:31 646572327
Что за с тем, что мы якобы живем в симуляции? Это очередная жвачка для быдла или что-то потенциально реальное?

Алсо, параллельные вселенные тоже пиздеж?
Аноним 10/04/21 Суб 20:35:37 646573328
>>646572
>Что за с тем, что мы якобы живем в симуляции?
Религия для атеистов.
Аноним 10/04/21 Суб 21:03:33 646579329
>>646551
не делай вид что не смог осилить абзац маленьких буковок полностью
Аноним 10/04/21 Суб 21:38:02 646586330
LunarThoriumcon[...].jpg 817Кб, 2100x1100
2100x1100
>>646560
Представь себе, большую часть информации о поверхности Луны и её составе мы знаем как раз на основе данных с телескопов. Не земных, конечно, там разрешение слишком хуёвое, а с лунных спутников типа https://en.wikipedia.org/wiki/Lunar_Prospector и Кагуи.

Грунт и камни, конечно, тоже важны, и по ним можно узнать геологическую историю Луны, посмотреть на состав керна по вертикали, измерить выветривание пород, изучить пригодность грунта для строительства, и т.д., но чисто элементный состав прекрасно и через телескоп узнаётся.
Аноним 10/04/21 Суб 21:40:03 646587331
>>646586
Ты понимаешь разницу между химическим и минеральным составом?
Аноним 10/04/21 Суб 22:17:12 646590332
Если пернуть в космосе, какого типа двигателем будет срака?
Аноним 10/04/21 Суб 22:24:41 646592333
>>646587
Отличить каменистую планету от газового гиганта по спектру элементарно.

>>646590
Cold gas thruster на смеси углекислого газа, водорода, азота и прочих газов.
Аноним 11/04/21 Вск 03:59:18 646614334
Аноним 11/04/21 Вск 05:55:26 646615335
>>646592
> спектру элементарно.
В теории, а на практике никто не наблюдал спектральные линии атмосфер экзопланет.
Аноним 11/04/21 Вск 08:15:04 646620336
Если в космосе, на орбите Марса, надуть воздушный шар радиусом 100 метров, без обогревателей и охладителей, какая температура воздуха будет внутри?
Аноним 11/04/21 Вск 08:50:50 646622337
Аноним 11/04/21 Вск 09:35:09 646624338
>>646620
Не такой простой вопрос кстати как кажется.
Аноним 11/04/21 Вск 09:45:35 646630339
Аноним 11/04/21 Вск 10:26:47 646635340
Что дороже - твердотопливный ускоритель Шаттла или советская жикдосктная Энергия?
Аноним 11/04/21 Вск 10:27:18 646636341
>>646630
С чего ты так решил?
Аноним 11/04/21 Вск 10:32:58 646639342
Аноним 11/04/21 Вск 10:35:55 646640343
>>646635
Жидкостная конечно. Твердотопливные ракеты это дешевая хуйня, там ничего сложного и дорогого нет.
Аноним 11/04/21 Вск 10:48:32 646641344
>>646640
>там ничего сложного и дорогого нет.
А тысячи тонн перхлората аммония, связующего, горючего, корпус ускорителя, камеры сгорания?
Аноним 11/04/21 Вск 10:51:23 646643345
>>646641
Это самое дорогое, но по сравнению с нормальной ракетой это наверное копейки
Аноним 11/04/21 Вск 10:54:01 646644346
>>646643
>наверное
Так ты гадалка?
Аноним 11/04/21 Вск 11:03:20 646648347
>>646495
>астрономическая теория
>чтобы не было всякой ебли и расчетов
Одно выбери
Аноним 11/04/21 Вск 11:10:40 646649348
>>646644
Ну это ж логично просто
Аноним 11/04/21 Вск 11:10:56 646650349
>>646648
>чтобы не было всякой ебли и расчетов
Вот это.
Аноним 11/04/21 Вск 11:14:49 646652350
может ли быть разумная человекоподобная жизнь на приливнозахваченных планетах? во-первых, как на таких планетах с полем? во-вторых, есть ли там смена времён года? не, ну смена дня и ночи тут понятно отсутствует, а вот как насчёт наклона оси смены сезонов из-за наклона оси? или приливка и тут сконтрит наклон?

каковы перспективы у разумной (человеноподобной) жизни на твёрдых поверхностях ледяных планет-лун? ну, типа, как ганимед, только подо льдом и на дней многокилометрового океана, то есть на поверхности самой планеты, а не на её ледяной корочке
Аноним 11/04/21 Вск 14:18:26 646680351
Почему не используют лазеры в качестве ПВО?
Мощности не хватает? Атмосфера мешает?
Аноним 11/04/21 Вск 14:24:24 646684352
Аноним 11/04/21 Вск 14:24:41 646685353
>>645048
Мощная установка на термояде/антиматерии в точке Лагранжа
Аноним 11/04/21 Вск 14:40:21 646691354
Аноним 11/04/21 Вск 14:41:55 646693355
>>646691
И лазером за горизонт не выстрелишь
Аноним 11/04/21 Вск 15:21:33 646701356
>>646691
>дешевле
Ага, скажи это какому-нибудь THAAD за 3 млрд $.

Необходимость в том, что лазер точнее гораздо и быстрее.
Аноним 11/04/21 Вск 15:59:48 646704357
>>646701
У лазера есть большой минус - рассеяние луча. Обычная дымовуха защитит огромный кусок неба от лазера, а если рядом с установкой использовать, то лазер выведен из боя. Так же отражающее покрытие боеголовки/кабины, хотя бы белой краской - сильно снизит мощность нагрева, не говоря уже о чем-то типа катафота. Да хотя бы облачная погода или дождик и лазер становится бесполезен.
Аноним 11/04/21 Вск 16:20:17 646706358
gg
Аноним 11/04/21 Вск 18:10:23 646725359
>>646572
>Это очередная жвачка для быдла
Да.
>Алсо, параллельные вселенные тоже пиздеж?
Наверняка никто не знает, но, скорее всего, нет.
Аноним 11/04/21 Вск 21:23:07 646737360
>>646652
В Библии написано что Бог создал Землю за 7 дней. Речь о человеке идет в контексте одной Земли.

О других планетах речи не шло, так что скорее всего нет.
Аноним 12/04/21 Пнд 04:14:26 646766361
>>646572
>Что за с тем, что мы якобы живем в симуляции? Это очередная жвачка для быдла или что-то потенциально реальное?

Гипотеза. Вполне себе научная, но гипотеза.
Аноним 12/04/21 Пнд 04:17:00 646767362
>>646586
> мы знаем как раз на основе данных с телескопов

А как выяснить изотопный состав по телескопу? Как обнаружить наличие льда под грунтом по телескопу? Как выяснить состав черной жижи на Меркурии по телескопу?
Аноним 12/04/21 Пнд 04:23:32 646768363
>>646652
В какой-то книжке читал что не так они страшны по последним расчётам, эти приливозахваченные планеты. Насчет разумной и человекообразной - вопрос, но жизнь в принципе теоретически на них возможна.
Аноним 12/04/21 Пнд 05:06:42 646769364
>>646768
>гипотетически на них возможна.
пофиксил
Аноним 12/04/21 Пнд 07:59:23 646781365
>>646652

Если планета захвачена приливным тяготением звезды, то это значит, что она - находится очень близко к звезде. А значит, речь идет про планету в системе красного или коричневого карлика.
Коричневый карлик - можно сразу вычеркнуть, они слишком холодны.
Проблема красного карлика - в нестабильности звезды. Но, можно допустить мысль, что планета обладает мощной магнитосферой.
В любом случае, это будет планета - непохожая ни на что. На одной стороне которой - вечный день. а на другой, вечная - ночь. На которой дуют мощнейшие ветры, а линия терминатора - будет удивительным местом жизни и смерти.

Чисто теоретически - может. Посмотреть бы на такой мир хоть одним глазком.
Аноним 12/04/21 Пнд 08:05:23 646782366
>>646652

Да, еще стоит сказать, что - скорее всего, приливные силы звезды будут активно деформировать ядро планеты.
Поэтому- активная тектоническая, вулканическая деятельность.
Как пример, Ио у Юпитера.
Аноним 12/04/21 Пнд 08:51:46 646784367
>>646766
>Вполне себе научная
Нефальсифицируемая, недоказуемая, ничего не предсказывающая. Охуеть научная.
Аноним 12/04/21 Пнд 08:54:25 646785368
>>646784
>недоказуемая
Это входит в определение научности? Я думал, что опровержимость - критерий.
Аноним 12/04/21 Пнд 08:58:53 646787369
>>646785
Да, симуляция в первую очередь неопровержима.
Аноним 12/04/21 Пнд 09:06:32 646789370
jopa
Аноним 12/04/21 Пнд 09:19:16 646791371
Простит ли нас Юра Небоходец за то, что мы всё проебали?
Аноним 12/04/21 Пнд 09:33:30 646793372
Гуманитариеблядь итт. Задумался сейчас об энтропии, если все процессы в природе идут в сторону увеличения энтропии, то правильно я понимаю, что мы можем с большой вероятностью предположить, что будет со вселенной? То есть всё будет разрушаться, до последней частицы, пока не останется тупо пустое пространство, бесконечно расширяюшиеся где уже никогда ничего не происходит произойдёт? Я правильно понимаю? Просто разные версии о конце вселенной, схлопывание, разрыв материи и т.п поебота. Может всё вот так, проще и логичнее?
Аноним 12/04/21 Пнд 09:46:08 646796373
>>646793
Рассуждения об энтропии пришли из термодинамики. Когда человечество освоит квантовую механику на прикладном уровне (хотя, судя по последним новостям, не "когда", а "если"), то энтропия пойдёт нахуй. Будем варить материю и антиматерию из вакуума. Раскладывать по чанам и заряжать межпространственные порталы.
Аноним 12/04/21 Пнд 10:44:12 646800374
>>646793
Почему все помнят про второй закон термодинамики, но никто не вспоминает про первый?
Аноним 12/04/21 Пнд 10:53:39 646801375
Что тяжелее: килограмм говна или килограмм варенья?
Аноним 12/04/21 Пнд 11:03:48 646804376
>>646801
>Что тяжелее
Если съесть, то тяжелее конечно же кило говна. А если высрать, то высрать варенье не переварив его в говно практически невозможно.
Аноним 12/04/21 Пнд 11:07:32 646807377
image.png 114Кб, 292x359
292x359
>>646801
Они оба килограмм.
Аноним 12/04/21 Пнд 11:07:53 646809378
>>646801
Межгалактический магазин:
- Мне пожалуйста кило чёрных дыр.
- Взвешиваем... Десять мегатонн получилось.
- Эх, ладно. Давайте.
Аноним 12/04/21 Пнд 11:27:43 646817379
Когда перестанут мучать мкс?
Аноним 12/04/21 Пнд 11:28:24 646818380
Аноним 12/04/21 Пнд 11:36:38 646821381
>>646817
А она мучается?
> сбросить в океан модульную структуру
Максимум ебанатство.
Запускать новые модули. Выводить и контролируемо сбрасывать старые. Не? Не! Ебанём в океан всю структуру!
Аноним 12/04/21 Пнд 11:47:24 646823382
>>646821
> новые
Задачи нашёл?
Аноним 12/04/21 Пнд 11:53:53 646825383
Чем ближе к экватору, тем меньшая тяга нужна ракете чтобы выйти на орбиту? А если старт точно с полюса?
Аноним 12/04/21 Пнд 11:58:10 646826384
>>646825

Старт с полюса почти на треть менее энергоэффективен нежели с экватора. Грубо говоря, одна и таже ракета выведет при запуске с полюса на 30% меньше полезной нагрузки.
Аноним 12/04/21 Пнд 11:59:41 646827385
>>646826
А старт против вращения Земли?
Аноним 12/04/21 Пнд 12:00:52 646828386
>>646826
Даже если на полярную орбиту?
Аноним 12/04/21 Пнд 12:01:00 646829387

>>646826
>Старт с полюса почти на треть менее энергоэффективен нежели с экватора. Грубо говоря, одна и таже ракета выведет при запуске с полюса на 30% меньше полезной нагрузки.

Суть в том, что при старте с экватора к скорости ракеты прибавляется скорость от центробежной силы вращения Земли?
А что если стартовать с полюса точно вертикально?
Аноним 12/04/21 Пнд 12:05:13 646830388
>>646827
Если задача вывести на орбиту груз, то если ты запустишь ракету против вращения, то скорость вращения земли будет работать против тебя же.
Иначе говоря, даже с полюса ее будет запустить эффективнее.
Исключение, это стратегические ракеты с ЯО, они расчитываются с учетом этого.
Аноним 12/04/21 Пнд 12:08:00 646833389
image.png 55Кб, 910x566
910x566
>>646829
>А что если стартовать с полюса точно вертикально?
Куда ты вертикально собрался? На полярную?
Аноним 12/04/21 Пнд 12:08:01 646834390
>>646829
Если ты построишь космодром строго на полюсе и запустишь ее вертикально, она взлетит вертикально.
Но запускать ракету с экватора все равно, эффективнее.
Даже в том случае, когда тебе нужно вывести спутник над северным полюсом. Тебе проще вывести спутник на экваторе, я затем сманеврировать в сторону полюса.
Тупо проще, дешевле и эффективнее.
Аноним 12/04/21 Пнд 12:23:16 646837391
>>646826
Смотря на какую орбиту. На полярную солнечно-синхронную, наоборот, чем ближе к экватору, тем меньше ПН, т.к. вращение Земли начинает работать против ракеты. В целом бонус от вращения Земли примерно пропорционален косинусу наклонения.

>>646834
Хуйню несешь, ничего подобного. Смена наклонения это невероятно дорого, а уж маневр на 90° настолько дорогой (требуемая дельта в √2 раз больше орбитальной скорости), что эффективнее будет затормозить в атмосфере, упасть на землю, и уже взлететь с земли на нужное наклонение.
Аноним 12/04/21 Пнд 12:43:22 646838392
>>646837

Метопы эурогейские, тем не менее, с Байконура запускали, а не с Плесецка.
Аноним 12/04/21 Пнд 13:04:01 646841393
>>646823
Датьпососать уже не задача?
Аноним 12/04/21 Пнд 13:04:26 646842394
Аноним 12/04/21 Пнд 13:14:19 646844395
>>646837
> что эффективнее будет затормозить в атмосфере, упасть на землю, и уже взлететь с земли на нужное наклонение
Или долететь до луны и поменять об неё наклонение. Хз как это правильно сказать вообще, но я так в ксп делал.
Аноним 12/04/21 Пнд 13:22:58 646845396
>>646801
В вакууме или атмосфере?
Аноним 12/04/21 Пнд 13:25:41 646847397
>>646841
Да, давно уже не задача.
Аноним 12/04/21 Пнд 13:26:06 646848398
>>646844
Да и без луны можно биэллиптиком.
Аноним 12/04/21 Пнд 13:38:23 646850399
Почему мир стыдиться признать заслуги Третьего Рейха в ракетостроение? Почему не празднуют дату достижения Фау-2 космического пространства? Да, полет не орбитальный, а суб-орбитальный, но это тоже веха, впервые что-то сотворенное рукой человека достигло космоса.
Мы должны быть выше предубеждений и ничего не замалчивать. Страх перед Гитлером только усиливает его, подкачивает ему ману.
Аноним 12/04/21 Пнд 13:41:52 646853400
>>646850
>Мы должны быть выше предубеждений
А мы нет. Вот поэтому и стыдимся. Да что там заслуги третьего рейха, сейчас вовсю плоскоземельщики числа наращивают, какого здравого смысла ты ожидаешь-то?
Аноним 12/04/21 Пнд 13:48:27 646855401
Эх, а при Гитлере сейчас бы уже на Марсе апфельбаумы цвели...
Аноним 12/04/21 Пнд 13:52:28 646856402
>>646796
> не "когда", а "если"
Что за последние новости?
Аноним 12/04/21 Пнд 14:55:18 646857403
>>646853
Это просто тралинг. Удивительно что кто-то ведется.
Аноним 12/04/21 Пнд 15:04:29 646858404
>>646857
В чем тралинг, вполне валидный вопрос. Троллинг ведут с целью обидеть кого-то, тут ничего обидного нет.
Аноним 12/04/21 Пнд 15:36:48 646861405
>>646858
Не обязательно обидеть. Как можно обидеть кого-то, утверждая, что Земля имеет форму цилинда, пирамиды или додекаэдра? Это просто ржака: наблюдать, как человек пытается опровергнуть рандомную дичь и не может.
Аноним 12/04/21 Пнд 15:47:24 646862406
Стикер 63Кб, 400x479
400x479
>>646861
Ну ладно, как скажешь.
Аноним 12/04/21 Пнд 15:54:20 646863407
>>646861
В чем тут тралинг? Разве гитлеровская Германия не первая в космосе?
Аноним 12/04/21 Пнд 16:29:24 646865408
Расскажите какие бывают орбиты? Я знаю что есть низкая орбита,когда тупо вокруг Земли летает, и геостационарная когда над одним местом Земли летит с той же скоростью что и Земля вращается. А еще есть полярная? И какие-то еще?
Аноним 12/04/21 Пнд 16:30:02 646866409
>>646863
Если уж быть честным, то микроб вылетевший на астероиде в архее - первый в космосе.
Аноним 12/04/21 Пнд 16:31:41 646868410
>>646863
Не только первая в космосе, но и базы на Луне и Марсе имеет.
Аноним 12/04/21 Пнд 16:32:33 646869411
Вопрос потенциально "дискуссионный".

Всё ли мы проебали?
Аноним 12/04/21 Пнд 16:41:19 646870412
image.png 210Кб, 3514x543
3514x543
Аноним 12/04/21 Пнд 16:44:21 646871413
>>646866
>>646868
Как же у некоторых бомбит от того, что Гитлер - первый в космосе.
Аноним 12/04/21 Пнд 16:46:20 646872414
>>646869
И если да, то простил бы фон Браун роскосых за это?
Аноним 12/04/21 Пнд 17:03:33 646875415
Аноним 12/04/21 Пнд 17:03:33 646876416
Аноним 12/04/21 Пнд 17:06:52 646877417
Аноним 12/04/21 Пнд 17:17:31 646880418
>>646781
>На одной стороне которой - вечный день. а на другой, вечная - ночь. На которой дуют мощнейшие ветры
а почему там должны дуть ветра? предположим такой приливный мир плоским как циферблат и рсположим звезду в направлении 12. значит, с 3 до 9 будет вечная ночь. как, по твоему долны тут дуть ветра и откуда они должны взяться, если возможное течение ветров должно быть так или иначе обусловлено давление света. но если свет давит сразу и с 3 (в направлении 4 и 5 часов), и с 9 (в направлении 8 и 7 часов), то получается ветре всегда дуют со всех сторон во тьму, и воздуху оттуда некуда уйти, значит, должно быть что-то вроде пресыщенного неветропригодного сектора ночной стороны - на вскидку от 3:30 до 8:30 полного ледяного штиля, а ветра должны иметь форму тороида, опоясывающего терминатор. на всю ночную сторону никак не тянет
Аноним 12/04/21 Пнд 17:21:53 646881419
>>646871
У меня не бомбит. Я только рад, что он жив и прилетит с Луны или с Марса, чтобы спасти человечество.
Аноним 12/04/21 Пнд 17:23:14 646882420
>>646737
В Библии также написано, что Господь Отец, Господь Сын и Господь Святой Дух создали человека по образу и подобию своему, и неизвестно сколько у этих терраформовщиков, взявшихся за проект "Земля", прошло подрядов на колонизации миров и сколько из них разрабатывали разумных существ по одному лекалу.
К тому же, в Библии же написано про лукавого, что он соблазнял всю вселенную и речь тут явно не о том, что хуем крутил посреди супервойда, потому что эксгибиционист-вакуумосексуал, а речь шла о многих других мирах и разумных, сооответственно, цивилизациях.
К тому же само описание нечистых духов показывает нам подобных жаб амфибий-рептилоидов, что свидетельствует о 100% факте существования не только инопланетян, но и нечеловекоподобных.
Аноним 12/04/21 Пнд 18:17:50 646888421
>>646880
> ветре всегда дуют со всех сторон во тьму, и воздуху оттуда некуда уйти

Наоборот же будет. Воздух на освещенной стороне терминатора будет нагреваться и подниматься вверх, в стратосферу и расползаться в стороны, и на тёмной стороне охлаждаться и опускаться вниз, и вдоль земли снова вытекать на солнечную сторону. Около поверхности ветер будет 100% времени дуть от холодной тёмной стороны к тёплой солнечной, т.е. образуется чё-то типа земной ячейки Хэдли.

Только на Земле циркуляция расположена в широтном направлении, а на приливно захваченной планете будет вдоль терминатора, т.е. образуется этакое сплошное кольцо ветров. На полюсах будет слабенько, т.к. там свет падает под небольшим углом и перепад температур маленький, а около экватора будет настоящий стоячий ураган.
Аноним 12/04/21 Пнд 18:20:07 646890422
>>646888
какие у тебя полюса в лоченной плонетке
Аноним 12/04/21 Пнд 18:26:07 646893423
>>646890
Приливный захват это не отсутствие вращения, а его синхронизированность с оборотами вокруг звезды. И ось будет, и полюса будут, и экватор, и даже полярная звезда. Тропиков вот не будет, правда.
Аноним 12/04/21 Пнд 18:29:31 646895424
>>646890
Хотя не, извиняюсь, действительно хуйню сказал. Полюса будут, но не будет температурного градиента вокруг терминатора, так что по всему кольцу ветер будет одинаковой силы.
Аноним 12/04/21 Пнд 18:41:26 646896425
>>646793
>То есть всё будет разрушаться, до последней частицы, пока не останется тупо пустое пространство, бесконечно расширяюшиеся где уже никогда ничего не происходит произойдёт?
Ну там не совсем ничего не будет происходить. Будут квантовые флуктуации. А так, в целом, да.
>Может всё вот так, проще и логичнее?
Именно. Только при условии, что темная энергия - это космологическая постоянная.
Аноним 12/04/21 Пнд 19:54:29 646909426
>>646893

Ключевое, это даже не приливный захват и не отсутствие смены дня и ночи. Звезда будет постоянно деформировать ядро и такой планеты будет очень высокая сейсмическая активность.
Упомянули Ио, ну вот оно самое. Масса действующих вулканов, высирающих в атмосферу массу "полезного" и полное отстутсвие спокойных мест, где жизнь может последовательно развиваться в течении долгого времени.
Очень маловероятно, что на такой планете жизнь сможет эволюционировать во что-то высокоорганизованное. Какие нить простейшие, может быть.
Так-что только желтые или оранжевые карлики.
Аноним 12/04/21 Пнд 20:01:53 646912427
А что с экзотическими звездами? У какой-нить нейтронной звезды, напрмер.
Аноним 12/04/21 Пнд 20:03:21 646915428
>>646912
Я про жизнь у нейтронной звезды. И известны ли еще какие-нить вариации жизни кроме белковой, допускаются ли варианты существования жизни по другим принципам. Хуй знает, кремниевая какая нибудь или железная.
Аноним 12/04/21 Пнд 20:12:00 646918429
>>646915

У нейтронной звезды не выживет даже двочер. Нейтронная звезда - это сраный ад, выжигающий всех и вся. Пофантазировать можно конечно.
Аноним 12/04/21 Пнд 20:15:48 646920430
7fbcfec1cee9f7e[...].jpg 86Кб, 1200x600
1200x600
Если в гипотезе о формировании Луны говорится что с Землей столкнулась планета размером с Марс и образовалась Луна+Земля, то какого размера была тогда Земля до столкновения с этой гиппотетической Тейей?
Аноним 12/04/21 Пнд 20:22:21 646921431
>>646912
>>646915
Если не вплотную к нейтронной звезде, тогда норм. Например, если поодаль от нейтроник врщается чёрный или коричневый карлик (суперЮпитер), на орбите которого вполне может вращаться очень даже уютный и комфортный мир. Даже по меркам углеродных белковых поверхностных, почти вакуумных человеков.
Аноним 12/04/21 Пнд 20:26:47 646923432
>>646909
> высирающих
жизнь же в таком кале и зародилась, не?
Аноним 12/04/21 Пнд 20:31:22 646924433
>>646920
> какая
чуть меньше была
Аноним 12/04/21 Пнд 20:35:50 646926434
>>646915
Химики говорят, что жизнь не на основе кремния или азота нестабильна и за фемтосекунду должна размножится, что очевидно невозможно. Из пригодных растворителей известно только вода и ряд веществ кипящих при низких температурах, что ввиду термодинамики сильно снижает шансы на жизнь.
О других основах жизни кроме углерода неизвестно, это будет либо какой-то очень редкий элемент, что очевидно снижает шансы, либо очень экзотические условия умеренно редкого элемента. С растворителем та же байда. Шансы что будет не вода и не углерод стремительно малы если вообще возможны.
Подробнее смотри лекцию https://www.youtube.com/watch?v=n79xVJur1Q8

И да у нейтронной звезды хана всему живому, если ты не защищён километрами камня. А если ты зарыт под землю(у которой ввиду истории вряд ли будет жизнепригодные недра), то и смысл? Получать энергию от нейтронки ты только не сможешь, если только не приливными силами. Опять же Нейтронные звёзды рождаются из больших звёзд, а большие звёзды живут мало и взрываются в сверхновой, если планеты и успеют образоваться, то остыть они точно не успеют. Если только приливным захватом.
Аноним 12/04/21 Пнд 20:40:03 646928435
>>646926
>последнее словосочетание
*гравитационный захват.
Аноним 12/04/21 Пнд 20:48:53 646929436
>>646923

Для зарождения жизни - судя по всему, необходимое условие.
Но вот развитию жизни - мешает оч сильно. У Земли был период такой активности, но это был именно период. Который закончился и жизнь могла спокойно эволюционировать.
Влияние вулканической деятельности на биосферу - огромно. Мощное извержение одного вулкана, способно понизить температуру на десяток градусов.
А прикинь, таких вулканов будет сотня и они будут непрерывно высирать в атмосферу ништяки на протяжении сотен миллионов лет.
Нет, без шансов для жизни, ИМХО.
Аноним 12/04/21 Пнд 20:51:35 646930437
>>646926
Вообще-то, возможны на основе обычного азота. Видел видеолекцию, где доказали, что при сильном давлении и низких температурах азот способен образовывать такие же сложные молекулярные структуры, какие может создавать углерод при земных условиях.
Это значит, что на Уране/Нептуне можно получит азот-основную жизнь. Впрочем, не повезёт ей с гравитационным колодцем родного мира, ой, не повезёт
Аноним 12/04/21 Пнд 20:55:29 646932438
>>646912
Нейтронная звезда - это финал жизни звезды, а рождается она, во вспышке сверхновой.
А вспышка сверхновой, это не хуй в стакане, а тотальная стерилизация всей солнечной системы, если она была, конечно. А еще, тотальная зачистка всей солнечной системы от всевозможного говна, как планет, так их атмосферы, всевозможных обьектов типа комет, которые могли бы хоть что-то занести на планеты.
Время жизни нейтронной звезды, довольно мало.
Так что нет, планета около нейтронной звезды (если она чудом уцелела) - будет абсолютно стерильна.
Аноним 12/04/21 Пнд 21:10:50 646933439
>>646930

Скорее на Титане. Но это будет ОЧЕНЬ ОЧЕНЬ экзотическая жизнь.
Аноним 12/04/21 Пнд 21:19:41 646936440
>>646930
Так-то да, но пока это не доказуемо.
>>646933
Та же углеродная жизнь, только с растворителем из метана.
Аноним 12/04/21 Пнд 21:32:21 646938441
>>646936

Но она - небелковая, вот в чем дело.
Аноним 12/04/21 Пнд 21:35:39 646939442
Академики и нобелевские лауреаты.
Подскажите.
Вот есть Лиго, который фиксирует всякие мощные гравитационные события.
Любопытным оказалось событие, в результате которого слияние двух ЧД произошло за секунду, а энергия которая ушла в гравитационные волны равна массе покоя 3 масс солнца.
Как бы за секунду, это овердохуя, насколько я понимаю.
А что будет с двочером если он окажется на расстоянии астрономической единицы от подобного события?
Разорвет нахуй знакопеременной гравитационной волной?
Аноним 12/04/21 Пнд 21:37:16 646940443
>>646938
белковая, максимум другие белки и жиры для холодного метана
Аноним 12/04/21 Пнд 21:39:58 646941444
>>646939
> Разорвёт
недолжно, в человеке жизненно важной сверх точной оптики нет
Аноним 12/04/21 Пнд 21:46:33 646942445
Зацените план:
1) Накачиваем Титан кислородом
2) Поджигаем
Аноним 12/04/21 Пнд 21:50:05 646943446
>>646933
На Титане не достаточно ебическое давление, чтобы азот смог организовавть ДНК-подобные суперсложные молекулы. КЛИМАТ НЕ ТОТ, так сказать.
Аноним 12/04/21 Пнд 22:05:42 646946447
>>646942
Доставка кислорода на титан потребует больше энергии, чем выделиться при горении его атмосферы.

Из-за низкой температуры и наличие в атмосфере непредельных углеводородов реакция просто не сможет самоподдерживаться.
Аноним 12/04/21 Пнд 22:13:07 646947448
>>646939
Действие гравитационной волны это приливные силы. Длина волны от твоих событий слишком большая по сравнению человеческой тушкой, ничего особенного не ощутит, даже если прям рядом с горизонтом будет.
Чтоб человека гравитационной волной распидорасило, нужно быть рядом с парой маломассивных(масса порядка Юпитера) ЧД, хотя его и обычном статическим приливом разорвет.
Аноним 13/04/21 Втр 06:08:01 646983449
>>646942
Отличный план, просто охуенный. Надежный, блять, как швейцарские часы.
Аноним 13/04/21 Втр 07:13:46 646985450
>>646947
Ну то есть планеты распидарасит, а человека - ни?
Аноним 13/04/21 Втр 07:31:36 646986451
Аноним 13/04/21 Втр 08:09:14 646992452
image.png 1739Кб, 1280x966
1280x966
image.png 579Кб, 750x592
750x592
Если на орбите подалеку от сливающихся ЧД разместить светящиеся маячки и смотреть на них с эклиптики издалека, мы увидим воочию "взрывную волну" гравитационных волн? Картинки рилейтед.
Аноним 13/04/21 Втр 08:43:39 646994453
>>646823
Заменять старые ламповые модули на новые нанотехнологические, как доски у корабля Тесея. Благодаря новым материалам больше радиусы = больше места. Ебануть кольцо из модулей, запустить вращаться и получить эмуляцию гравитации. Современные управляюще-вычислительные системы с болшей пропускной способностью для более эффективного майнинга протеинов научных изысканий. Задач куча.
Аноним 13/04/21 Втр 08:46:41 646995454
>>646994


Всего то делов, сделать старшип набитый мячками, найти две черных дыры, которые в ближайшее время сольются и ебануть к ним, используя гипердрайв.
Аноним 13/04/21 Втр 08:48:12 646996455
>>646995
Изи задачка для Гитлера с Марса.
Аноним 13/04/21 Втр 08:54:52 646997456
>>646995
Вопрос не в реализации, мы даже на марс огурцов послать не можем, вопрос гипотетический чисто.
Аноним 13/04/21 Втр 08:56:10 646998457
Хде иноплане тянки?
Аноним 13/04/21 Втр 08:59:18 646999458
>>646998
На других планетах.
Аноним 13/04/21 Втр 09:24:15 647002459
>>646998

Мы разделены не только расстоянием, но и временем.
То, что мы есть - как цивилизация, известно только в радиусе примерно 120 световых лет.
Аноним 13/04/21 Втр 09:26:54 647005460
>>646997
Гипотетические гравитационные волны подтверждены практически.
Так что задумка, даже если допустить, что мы, как цивилизация, способны осуществить подобный проект - бессмысленна.
Намного интересней попробовать использовать энергию вращающейся ЧД.
Вот це да.
Аноним 13/04/21 Втр 09:44:10 647008461
>>647002

В радиусе 100 световых лет. сотни звезд
Аноним 13/04/21 Втр 09:44:46 647010462
>>647005
Да не надо этот проект осуществлять, я спросил как это будет выглядеть? Маячки останутся на месте или покачнутся как тестовые линии при ядерном взрыве?
Аноним 13/04/21 Втр 09:48:19 647011463
>>647008
И все они мертвы. SETI не даст соврать.
Аноним 13/04/21 Втр 09:49:12 647012464
>>647010
> Маячки останутся на месте или покачнутся как тестовые линии при ядерном взрыве?
1. Покачнутся.
2. Останутся.
Аноним 13/04/21 Втр 09:55:09 647013465
>>647010
Ну это же искривление пространства времени, так что да, для стороннего наблюдателя они изменят свое положение.
Будут качаться, вверх и вниз
Аноним 13/04/21 Втр 09:58:55 647015466
>>647011
Или, как вариант, там обезьяны с копьями бегают и в радио еще не умеют.
Или умеют, но им глубоко похуй на радио.
Аноним 13/04/21 Втр 09:59:07 647016467
>>647013
>вверх и вниз
Какие еще "верх" и "низ"?
Аноним 13/04/21 Втр 10:00:16 647017468
image.png 130Кб, 400x400
400x400
>>647015
>2021
>не уметь в радио
Жалкая чернь.
Аноним 13/04/21 Втр 10:03:57 647018469
image.png 1016Кб, 895x505
895x505
>>647016

Относительно плоскости события, разумеется.
Аноним 13/04/21 Втр 10:04:52 647019470
>>647017
Кстати, это канешн лол, но то, что мы можем быть одной из самых развитых цивилизаций в округе, не так уж невозможно.
Аноним 13/04/21 Втр 10:18:21 647020471
>>647019
>гуляя по лесу, я могу оказаться самым умным человеком в округе
Очень глубокая мысль.
Аноним 13/04/21 Втр 10:30:11 647027472
Глянул тут Нила Дегрода Тайсона у Рогана, Штерна давно уже смотрел, но насколько я помню он тоже обозначал размеры черных дыр в солнечных массах. Почему часто можно встретить выражения, типа "черная дыра размером с Солнце" явно подразумевая диаметр дыры, вот только от чего и до чего измерять, диаметр горизонта событий или диаметр гравитационного влияния или диаметр сингулярностилол? Это журнашлюха или научпопер по глупости зафорсили такую хуйню, а идиоты за ними повторяют?
Аноним 13/04/21 Втр 10:35:29 647028473
>>647015
> и в радио еще не умеют
Астрономы говорят, что с помощью своих телескопов они могут улавливать малейшие отличия в спектральных линиях отражённого излучения экзопланет так, что даже в соседних галактиках могут распознать слишком много пукающие стада коров или слишком много дымящие заводы. Что уж говорить о 100 св.л. окрестности.
Нет. Там точно всё мертво.
И никто нам не поможет.
И. Не. Надо. Помогать.
Аноним 13/04/21 Втр 10:38:39 647029474
>>647027
> по глупости зафорсили
ЗЫС. Как орбитальную модель атома. Физики понимают, что такое атом, но объяснить не могут. Поэтому сдаются и обесняют на отъебись.
> ой, шарик карочи, шарик ядра как звезда, шарики электронов, как планеты. Ясно? Так и запишите.
Аноним 13/04/21 Втр 10:39:49 647030475
>>647027
Из контекста можно понять же. Если речь о ЧДЗМ то речь о массе, если о массивных или промежуточных, то о радиусе Шварцнеггера.
>>647029
Все могут, блядь, только ретарды даже модель с шариками понять не могут, какие вам электронные облака.
Аноним 13/04/21 Втр 10:46:47 647033476
tipichnyy-shkol[...].jpeg 46Кб, 720x610
720x610
>шарики
>облака
>понимают, но не понимают
Уровень экспертов итт.
Аноним 13/04/21 Втр 10:53:32 647034477
>>647033
В фундаментальном силовом поле поток электромагнитной энергии движется по геодезическим линиям согласно своей частоте колебаний. Эти геодезические линии образуют формы электронных облаков, например, для "электрона", который в общем-то не существует. Лично я для себя корпускулярно-волновой дуализм вычисляю в сторону волн. Частиц не существует, есть только волны и силовое воздействие волн, которые двигаясь по геодезическим линиям образуют замкнутые формы и передают таким образом силовые воздействия солового поля, что принято называть "частицами".
Аноним 13/04/21 Втр 10:54:34 647035478
>>647033
Ну вот и помалкивай, раз не понимаешь, пиздюк, нехуй тут срать.
Аноним 13/04/21 Втр 10:55:17 647036479
>>646946
>Доставка кислорода на титан потребует больше энергии, чем выделиться при горении его атмосферы.
Ну и что, зато как гореть охуенно будет!
Целую планету поджечь, такого ещё никто не делал!
Аноним 13/04/21 Втр 10:58:32 647038480
>>647028

Залупу за воротник они пока умеют.
Джеймса Уэбба запустят, тогда смогут рассмотреть экзопланеты в радиусе 10 (десяти) световых лет и считать их спектр.
Пока - нихуя не могут.
Пока, тока математика и то, крайне приблизительная.
Аноним 13/04/21 Втр 11:03:20 647039481
>>647038
Тем не менее в различных астроновостях постоянно пиздят, пр то как химический состав очередной экзопланеты до атома посчитали. Намекаешь им, что это пиздёж ради грантов - немедленно выхватываешь бан или мьют на стриме каком.
Аноним 13/04/21 Втр 11:04:44 647040482
>>646932

Тут еще не стоит забывать, что нейтронная звезда - финал жизни массивных и очень горячих звезд.
Так как они активны, то с них нормально так дует. Так дует, что сдувает все к ебеням в радиусе сотни астрономических единиц.
Там в принципе маловероятно образование планетной системы, а срок жизни такой звезды - мал.
Так что даже если допустить мысль о том, что есть планета примерно земной массы в обитаемой зоне такой звезды, жизнь тупо не успеет развиться из первичного бульона.
А дальнейшая вспышка сверхоновой, просто выжигает все к ебаням.
То, что не сдохнет в этом случе, добьет нейтронная звезда.
Короче, без шансов.
Аноним 13/04/21 Втр 11:06:29 647041483
image.png 326Кб, 591x422
591x422
>>647036
>Целую планету поджечь, такого ещё никто не делал!
В этой солнечной системе. Пока что.
Аноним 13/04/21 Втр 11:08:21 647042484
>>647039

Как только посчитают, сразу нобелевку дадут.
За Траппист всему коллективу провели вялым по губам, хотя работа была проделана - мое почтение.
Опять же, в случае с Траппистом, все выкладки были достаточно умозрительные, хотя и обоснованные, поскольку планеты находятся довольно близко друг к другу и заметно влияют гравитационно на соседей.
Но, один хуй, неясен ни состав атмосферы, ни состав планеты. Можно только вычислить примерную плотность и из этого сделать приблизительное заключение.
Прямое наблюдение экзопланеты сейчас недоступно.
Аноним 13/04/21 Втр 11:09:02 647043485
>>647040
> без шансов
А если жизнь разовьётся в каком-нибудь внутреннем океане?
Аноним 13/04/21 Втр 11:11:05 647044486
>>647042
> Прямое наблюдение
Никто о прямом наблюдении даже и не заикался. Речь шла о радиоволнах и плавно перешла на спектральные гадания, на которые нынче все хомячки надрачивают, а мочёные с них донаты стригут.
Аноним 13/04/21 Втр 11:12:28 647045487
>>647043
Крабовидная тумманость. В центре ее пульсар.
Скорость разлета 1500 км/с спустя 1000 лет. Ее размер уже 11 световых лет и все увеличивается.
Звезда вьебывает так, что может превысить яркость всех звезд в Галактике. Не уцелеет ничто.
Аноним 13/04/21 Втр 11:14:11 647046488
>>647044
Речь шла по стада пердящих коров на планете в другой галактике. Что бы разглядеть такие пуки, нужно непосредственное наблюдение за атмосферой планеты. Увидеть в спектре эмиссионные линии пердежа.
Аноним 13/04/21 Втр 11:21:04 647049489
>>647046
Не нужно. Разберись с матчастью.
Аноним 13/04/21 Втр 11:22:54 647050490
>>647049
Рассказывай поскорее.
Аноним 13/04/21 Втр 13:37:26 647066491
>>647018
Тока это событие не в 2х мерном пространстве происходит, а в 3х мерном. Это не поверхность воды, в которую кирпич упал.
Гравитационные волны распространяются не по плоскости, а во все стороны пространства.
В любом случае, полагаю что искажение пространства будет заметно, поскольку маяки должны будут изменить свои координаты около события, а затем, вернутся назад.
Волна, она все равно волна, даже такая.
Аноним 13/04/21 Втр 13:40:30 647067492
>>647066
А как это будет выглядеть? Фронт "сжатого пространства", или как это описать, пик волны, сделает маяк притянуться к эпицентру, а потом низкая часть волны сделает его оттолкнуться?
Аноним 13/04/21 Втр 13:43:07 647068493
>>647067
Да хуй его знает.
Нужно компьютерное моделирование делать.
Аноним 13/04/21 Втр 14:45:36 647072494
3 солнечных массы в состоянии покоя, это как то сильно дохуя.
где-то 4х1049 Вт. И эта мощность, которая была выделена в виде гравитационных волн в долю секунды.
Блять, это реально много.
Распидарасит все обьекты в радиусе нескольких световых лет.
Аноним 13/04/21 Втр 14:52:29 647073495
>>647072
>И эта мощность, которая была выделена в виде гравитационных волн в долю секунды.

И скорее всего это единицы процента от всей выделившийся энергии, а основная часть как обычно улетела в виде нейтрино.
Аноним 13/04/21 Втр 15:18:08 647075496
>>647072
Так гравитационное взаимодействие оно слабое самое, на 25 порядков слабее электромагнтиного.
Аноним 13/04/21 Втр 15:18:45 647076497
>>647075
Ой. На 25 слабее слабого, а электромагнитного на 36 порядков.
Аноним 13/04/21 Втр 17:57:51 647088498
>>647075

Именно по этой причине и пиздец.
Гравитационная энергия волн пары Солнце-Земля, дай бог памяти, около 6 или 7 кВт.
А тут, энергия гравитационной волны на 46 порядков выше. Это просто адское гравитационное возмущение метрики. Чудовищное блять. Сравнимое с энерговыделением звезд наблюдаемой части Вселенной.
Аноним 13/04/21 Втр 18:04:29 647089499
Аноним 13/04/21 Втр 18:09:53 647090500
image.png 319Кб, 600x578
600x578
>>644253 (OP)
Вкатился. Какого хуя картинка как в прошло... Падажжии...
Аноним 13/04/21 Втр 18:34:09 647097501
>>647089
Ну что, всё шутки шутишь?
Аноним 13/04/21 Втр 18:57:48 647100502
>>647097
норм перекат, хуле ебало скрючил?
Аноним 13/04/21 Втр 19:43:05 647108503
>>647100
Нормальный перекат будет ближе к выходным?
Аноним 13/04/21 Втр 20:22:13 647115504
image.png 327Кб, 600x600
600x600
>>647108
Нормальный перекат будет когда Илон пилотируемый старшип запустит.
Аноним 13/04/21 Втр 20:27:22 647116505
>>647088

Не, это гравитационная пара Солнце-Юпитер.
Солнце-Земля - 200 Вт.
Аноним 13/04/21 Втр 20:30:15 647117506
>>647116
Ебаный стыд. Коллапсируйте поле Хиггса в вакуум, такой обсер терпеть не надо.
Аноним 13/04/21 Втр 20:33:14 647120507
>>647117
Анус себе заколлапсируй, пёс.
Аноним 13/04/21 Втр 20:34:46 647121508
>>647115
тред без перекатов будет тонуть около года, у илона неиронично есть шанс запустить бочку с огурцом
Аноним 13/04/21 Втр 20:38:42 647123509
Перекатился... Бля, в пространственную аномалию кажись попал.
Аноним 13/04/21 Втр 20:40:27 647124510
>>647120
>Анус себе заколлапсируй, пёс.
- послышались гравитационные возмущения из другой галактики.

Но пацаны в ЛИГО как обычно не обратили внимания на эти возмущения. Пусть возмущается, что с нее взять? Гравитация - слабейшее взаимодействие и в ближайшее время о нем сделают открытие. Про нее уже в течение пяти столетий было написано уйма трудов и теперь эта тема была разработана настолько, что с ее помощью хотят надуть пузырь альбукерки.
Аноним 13/04/21 Втр 20:48:01 647125511
>>647124
че по пузырю кста, экзотику так и не искали?
Аноним 13/04/21 Втр 20:52:12 647127512
Аноним 13/04/21 Втр 21:40:11 647139513
>>647124

Не в этом столетии.
Запомните этот твит.
Аноним 13/04/21 Втр 21:43:12 647142514
>>647127
неиронично захватывающая литература, безудержный пессимизм во вступлении при описании проблематики вплоть до вылезающей причинности и временных петель резко переходящий в суть статьи, просто охуенно
Аноним 13/04/21 Втр 21:43:34 647143515
>>647139
Хуй знает, мне кажется, что в ближайшие лет десять надрочат ИИ выводить гипотезы и какая-нибудь жестянка выведет ебаную рабочую модель унифицирующей теории.
Аноним 14/04/21 Срд 09:12:28 647184516
Почему в России нет частных космических компаний?
Аноним 14/04/21 Срд 09:13:05 647185517
Аноним 14/04/21 Срд 12:30:33 647211518
0153.jpg 195Кб, 1000x650
1000x650
Когда уже запустят эту йобу? И когда ее запустят и включат, мы узнаем/увидим реально что-то новое и интересное? Или это будут ничего незначащие для простого обывателя циферки и плюс лишнее кило пикселей на снимочках черных дыр?
Аноним 14/04/21 Срд 12:34:38 647213519
>>647211
В конце октября. Да. Да.
Аноним 14/04/21 Срд 12:48:41 647216520
>>647213
Что да? Ничего интересного не будет? Или будет что-то интересное?
Аноним 14/04/21 Срд 12:52:43 647217521
>>647216
Будет что-то интересное. Но оно ничего не будет значить для простого обывателя.
Аноним 14/04/21 Срд 13:41:24 647222522
>>647211

Йоба уже под 10 миллиардов выходит.
Причем нихуя не в хрювнях.
Аноним 14/04/21 Срд 16:44:18 647232523
Что описывать в НФ-романе про другую планету? Уровень твердоты хочу сделать повыше. Очень хочу экзотическую планету, а не просто инопланетную Землю. Сюжет в целом есть, осталось придумать мир, завязать к нему пару оставленных неувязок и верёд
Аноним 14/04/21 Срд 16:46:39 647233524
>>647232
опиши большой титан в местами кислотных цветах, твёрже некуда будет
Аноним 14/04/21 Срд 16:49:17 647234525
Что за круглая хуитка на конце панелей прогресса? Вот тут в видео хорошо ее видно https://youtu.be/ra8nw7cVRUQ
Аноним 14/04/21 Срд 17:38:10 647241526
>>647234
Гидрамуфта, которая раскладывает панели.
Аноним 14/04/21 Срд 18:25:05 647254527
>>647211
>И когда ее запустят и включат, мы узнаем/увидим реально что-то новое и интересное?
Должно быть много новых данных по космологии и по всяким астрофизическим явлениям типа квазаров, экзопланет, нейтронных звезд, черных дыр и т.д.
Аноним 14/04/21 Срд 18:49:50 647259528
>>647216

Сможет напрямую наблюдать экзопланеты в радиусе 10 световых лет, сможет считать спектр планет.
Аноним 14/04/21 Срд 20:06:57 647278529
>>647232
Обитаемые спутники вокруг газового гиганта. Чтобы можно было летать между ними за считанные часы при удачном расположении. Джва года жду такую фантастику.
Аноним 14/04/21 Срд 20:11:53 647280530
>>647232

Мертвая планета, на которой есть следы некогда разумной жизни, вращающаяся вокруг пульсара.
Планета, у тусклого коричневого карлика, на которой царит вечный сумрак
Аноним 15/04/21 Чтв 09:22:23 647348531
>>647232
> Что описывать в НФ-романе про другую планету?
Сделой планету, которая ходит по орбите-восьмёрке между двумя солнцами двойной системы. Одно из солнц горячее, второе прохладнее. Когда планета подходит к горячему - наступает эпоха огня, все чупакабры прячутся от жары в подземные убежища. Когда планета подходит к прохладному - наступает эра льда, все чупакабры снова прячутся в подземные убежища, но уже от мороза. И только в момент перехода между солнцами есть короткий период комфортной биосферы, когда вся природа оживает, расцветает и чупакабры сладострастно размножаются.
Аноним 15/04/21 Чтв 10:27:18 647353532
15328529886960.jpg 687Кб, 1121x900
1121x900
>>647348
Так, абажжи.
С такой траекторий эпоха льда будет не у более холодного солнца, а в промежутке между солнцами. Говорят, что зашкаливающего, подавляющего числа звёздных систем, местное солнце всегда кратное (как минимум есть второе, но можно и больше солнц). Говорят, и у солнца есть вторая звезда, просто она тусклая и маленькая, к тому же по меркам солнечной системы, она далеко. Если предположить, что у нас есть планета с орбитой-восьмёркой, то не факт, что будет именно восьмёрка - тут и просто оооооооочень вытянутый овал вокруг двух звёзд сойдёт.
Если же мы преположим, что у нас два солнца достаточно близко друг к другу, то вря ди эра льда будет возможна под небосклоном, опаляемым двумя близкими солнцами.
Аноним 15/04/21 Чтв 10:34:36 647355533
>>647353
Нуу... Пусть анон-писатель проконсультируется в википедии. Я не большой уж и спец в астрономии, так что поправки принимаю.
Аноним 15/04/21 Чтв 10:37:22 647356534
>>647353
>Говорят, и у солнца есть вторая звезда, просто она тусклая и маленькая
Говорят, что кур доят.

>>647348
>Сделой планету, которая ходит по орбите-восьмёрке между двумя солнцами двойной системы.
Слишком нестабильная орбита, планета наебнётся до того как там жизнь многоклеточная появится
Аноним 15/04/21 Чтв 10:44:45 647358535
>>647356
Ну это ж фантастика. Даже в твердой можно научно опереться на теорию вероятностей и утверждать, что вот в этом очень маловероятном случае планета с орбитой восьмёркой успешно существует уже охулиард лет (удачно корректировавшаяся астероидами) и там жизнь успешно развилась.

Но вообще конечно, важна в моём посте идея о трёх эпохах. Орбитой восьмёркой можно и пожертвовать.
Аноним 15/04/21 Чтв 10:57:51 647362536
image.png 37Кб, 337x266
337x266
>>647348
>>647353
Орбит-восьмерок не может быть.
Зато "сезонность" можно обеспечить с двумя планетами меняющими орбиты как Эпиметей и Янус себе дерни пес https://youtu.be/6_Kq1EUkwqc
Аноним 15/04/21 Чтв 10:59:13 647363537
>>647358
>Орбитой восьмёркой можно и пожертвовать.
Если нужно что бы эпохи были прям очень долгими, а не в масштабе года, то планету поставить вращаться вокруг карликовой звезды, но так чтобы её тепла не хватало. И запустить этот цирк вокруг крупной звезды. При полном приближении к ней будет OCHE жарко, при полном удалении выпадение атмосферы в виде снега. В промежутке норм температуры.

>Даже в твердой можно научно опереться на теорию вероятностей и утверждать, что вот в этом очень маловероятном случае
Так лучше не делать, можно сразу тогда маловероятную звезду придумать, которая сама включается-выключается раз в тысячу лет
Аноним 15/04/21 Чтв 11:01:03 647364538
1614632806534.gif 897Кб, 800x430
800x430
>>647356
>Говорят, что кур доят.
Если кур не доят, то откуда птичье молоко? Шах и мат, атеисты!
Аноним 15/04/21 Чтв 11:03:36 647366539
>>647363
>Если нужно что бы эпохи были прям очень долгими, а не в масштабе года, то планету поставить вращаться вокруг карликовой звезды, но так чтобы её тепла не хватало. И запустить этот цирк вокруг крупной звезды. При полном приближении к ней будет OCHE жарко, при полном удалении выпадение атмосферы в виде снега. В промежутке норм температуры.
Ты только что Игру Престолов
Аноним 15/04/21 Чтв 11:05:10 647367540
image.png 24Кб, 1147x387
1147x387
Аноним 15/04/21 Чтв 11:05:42 647368541
>>647366
В игре престолов это из-за очень сильного колдунства
Аноним 15/04/21 Чтв 11:05:48 647369542
1324950744286.png 5Кб, 274x242
274x242
>>647364
Так не кур доят, а голубей.
https://ru.wikipedia.org/wiki/Зобное_молоко
>Зобное молоко, или «птичье молоко», или голубиное молоко, — творожистый секрет, вырабатываемый у некоторых птиц для кормления птенцов. Оно характерно для представителей семейства голубиных, ряда попугаев, фламинго и некоторых пингвинов
ШАХ И МАТ И УНО И ЯТЦИ
Аноним 15/04/21 Чтв 11:08:30 647370543
>>647362
Бриллиант! Я голосую за это.
>>647363
> планету поставить вращаться вокруг карликовой звезды, но так чтобы её тепла не хватало. И запустить этот цирк вокруг крупной звезды. При полном приближении к ней будет OCHE жарко, при полном удалении выпадение атмосферы в виде снега. В промежутке норм температуры.
Это уже черезчур.
> маловероятную звезду придумать, которая сама включается-выключается раз в тысячу лет
Покрытая остывшей коркой неразорвавшая в сверхновую звезда, у которой внутри нейтронное ядро, а в корке - дырка. И когда планета проходит мимо дырки, она освещается живительным рентгеном. Не, всё равно черезчур.
Аноним 15/04/21 Чтв 11:10:21 647371544
>>647369
То есть птичье молоко это на самом деле подзобный творожок?
Аноним 15/04/21 Чтв 11:10:26 647372545
>>647367
Я и по тексту понел. Но спасибо. А воще я несколько лет аутировал в Space Engine и всего этого навидался.
Аноним 15/04/21 Чтв 11:13:23 647373546
>>647368
Что мешает сильным колдунствам и их проекциям на планете Игры Престолов в виде нашествий дроконiв и ледозомби коррелировать и напрямую опираться на них, непосредственно завися от астрономические события в этой системе их влияния на все аспекты мира, включая и колдуно-волшебный?
Аноним 15/04/21 Чтв 11:15:00 647374547
>>647370
>неразорвавшая в сверхновую звезда
>хард сайфай
Приятель, ты чего, ей-богу. Либо чего-то навыдумывал, либо знаешь чего крутое, а не рассказываешь, поделись публикациями про неразорвавшиеся в сверхновые звезды и дырявые нейтронки. Вообще если хочешь эпизодически планеты рентгенить, просто положи нейтронку на бок и она будет из полюсов хуячить.
Аноним 15/04/21 Чтв 11:22:08 647376548
>>647373
Двачую, игропрестольные колдуны в таком случае астрономы-шарлатаны, которые зная, что похолодает, говорят такие, "вот мы заколдовали вам земли, что на них похолодает, платите нам дань, и тогда мы может быть через N лет сжалимся и расколдуем". И неграмотное быдло такое "НУОК".
>>647374
> ты чего, ей-богу
Я не такой любитель твердача, как реквестировавший анон. Законом не запрещено - выдумываю как хочу.
Аноним 15/04/21 Чтв 11:35:13 647377549
>>647373
То что по лору раньше нормально все было, а потом черная магнитя все испортила
Аноним 15/04/21 Чтв 11:48:50 647381550
>>647377
Раньше система вращения двух звёзд и одной планеты были сбалансированы, а потом какой-то массивный объект внёс небольшое изменение в баланс и понеслась пизда по кочкам
Аноним 15/04/21 Чтв 12:05:28 647384551
>>647376
>Законом не запрещено - выдумываю как хочу.
Тебе дай волю - ты и полые планеты выдумаешь и восьмерочные орбиты и звук в космосе. Фантазия это хорошо, малаца, но нормальный сайфач такие выдумки не выдержит.
Аноним 15/04/21 Чтв 13:34:37 647394552
Поясните что ограничивает длину волн? Почему спектр не может быть побольше туда или сюда? А?
Аноним 15/04/21 Чтв 13:38:45 647395553
>>647394
>Поясните что ограничивает длину волн?
Ничего.
>Почему спектр не может быть побольше туда или сюда?
Может.
>А?
Б.
Аноним 15/04/21 Чтв 14:03:49 647399554
>>647395
А чё на картинках он ограничен? Что находится за гамма волнами?
Аноним 15/04/21 Чтв 14:12:24 647400555
>>647399
>А чё на картинках он ограничен?
Как ты себе представляешь бесконечную картинку?
>Что находится за гамма волнами?
Гамма-волны. Все фотоны выше 100кэВ - гамма-излучение.
Аноним 15/04/21 Чтв 14:17:01 647403556
>>647400
Получается чем дальше - тем больше энергии. Значит где-то там есть такие значения, что выпустив эту волну она может подействовать на кирпичную стену как пуля, например?
Аноним 15/04/21 Чтв 14:21:14 647404557
>>647403
Как пуля с распидорашиванием - нет, один фотон такого не сделает. Всем атомам по пути электроны посбивает, но по пути их будет мало. Если в атом попадет (не попадет) - то расщепит, а его осколки на атомарном уровне покоцают окружающие.
Только мегаэлектронвольт это пиздецки мало, воздух вокруг наносит больше ущерба кирпичной стене, чем такой фотон.
Аноним 15/04/21 Чтв 14:46:09 647408558
>>647403
>она может подействовать на кирпичную стену как пуля, например
Даже относительно низкоэнергетические гамма фотоны проходят через стену насквозь.
Аноним 15/04/21 Чтв 14:47:21 647409559
>>647395
>Ничего.
Длина волны может быть меньше планковской длины?
Аноним 15/04/21 Чтв 14:52:16 647412560
>>647409
Может, но там уже включается другая физика и такие вещи пропадают из нашей реальности.
Аноним 15/04/21 Чтв 14:52:38 647413561
Аноним 15/04/21 Чтв 16:26:21 647417562
>>647348
Уже было в Симпсонах в Задаче трех тел Лю Цсизина
Аноним 15/04/21 Чтв 17:36:47 647423563
Что было бы, если на орбите Юпитера все луны заменились на Ганимеды, как бы это повлияло на систему?
А что будет, если вместо Ганимеда там будет Марс?
А что будет, если в Л4 и Л5 Юпитера поместить по Ганимеду?
А если по Марсу?
А если по Земле?
У какого из предложенных в вопросах выше возможны магнитное поле, жидкая вода и жизнь?
Аноним 15/04/21 Чтв 17:47:53 647425564
>>647423
> Что было бы, если на орбите Юпитера все луны заменились на Ганимеды, как бы это повлияло на систему?
все это сколько, их там десятки
на систему никак заметно не повлияет, л4-5 нестабильны на сотнях миллионов - миллиардах лет, масштабе формирования планет
> У какого из предложенных в вопросах выше возможны магнитное поле, жидкая вода и жизнь?
ни у кого, разве что подлёдная, на орбите юпа нужен мощный парник, он за снежной линией
Аноним 15/04/21 Чтв 18:41:43 647430565
>>647423
>Что было бы, если на орбите Юпитера все луны заменились на Ганимеды, как бы это повлияло на систему?
Часть выбросило, остальные вошли бы в резонанс.
>А что будет, если вместо Ганимеда там будет Марс?
Тоже самое.
>А что будет, если в Л4 и Л5 Юпитера поместить по Ганимеду?
>А если по Марсу?
>А если по Земле?
В этих точках, чем тяжелее объект, тем выше шанс что его выкинет или ебнется об Юпитер.
>У какого из предложенных в вопросах выше возможны магнитное поле, жидкая вода и жизнь?
Магнитное поле у любого достаточно тяжелого тела может быть, главное чтоб было проводящее ядро с конвекцией. Даже у Марса появилось бы поле, если он вращался быстрее или у него был достаточно массивный спутник.
Для жидкой воды нужна температура соответствующая.
А для жизни нужен ГОСПОДЬ БОГ
Аноним 15/04/21 Чтв 18:46:42 647432566
>>647430
>ебнется об Юпитер
Разве есть такой шанс? Почему? Для этого же надо траекторию в него иметь.
Аноним 15/04/21 Чтв 18:50:34 647434567
-.jpg 114Кб, 800x800
800x800
Почему мы считаем, что вселенная расширяется? На основании каких наблюдений и в какое время и в каком кластере космоса были сделаны измерения? Может, это не вселенная расширяется все, а лишь один какой-то кластер её, в котором мы и наблюдали расширение, экстраполировав его на всю вселенную?
Аноним 15/04/21 Чтв 18:59:45 647435568
>>647434
>Почему мы считаем, что вселенная расширяется?
Тому що эт заметно.
>На основании каких наблюдений и в какое время и в каком кластере космоса были сделаны измерения?
На основании красного смещения удаленных объектов.
>Может, это не вселенная расширяется все, а лишь один какой-то кластер её, в котором мы и наблюдали расширение, экстраполировав его на всю вселенную?
Везде наблюдается.
https://ru.wikipedia.org/wiki/Космологическое_красное_смещение
Аноним 15/04/21 Чтв 19:29:35 647443569
>>647432
л4 и л5 прям на орбите находится.
Рядом летает Сатурн, который возмущает орбиту, но за счет его резонанса с Юпитером возмущение снимается.
Когда у тела большая масса, то сила возмущения растет, а противодействие вызванное резонансом слабеет. В итоге возмущение постоянно "накапливается", что ведет к сходу с орбиты Юпитера. Ну дальше дело времени.
Аноним 15/04/21 Чтв 19:47:46 647444570
>>647443
Сраный Сатурн! Надо его об юпитер уебать, истинно говорю вам.
Аноним 15/04/21 Чтв 20:08:26 647447571
>>647444
Ты опоздал на пять лярдов лет.
Аноним 15/04/21 Чтв 21:53:53 647455572
>>647371
С сахаром и желатином.
Аноним 15/04/21 Чтв 22:42:49 647460573
Что там по детектору космического излучения, фотографирующий земную атмосферу?
Аноним 15/04/21 Чтв 23:24:20 647467574
>>647460
Какому из? Вроде недавно открыли гамма всплески и взрыв жидовской бомбы.
Аноним 16/04/21 Птн 06:42:43 647495575
>>647467
Тот который фотографирует половину поверхности земли, и по излучению из атмосферы регистрирует самые энергичные частицы.
Аноним 16/04/21 Птн 09:05:11 647503576
>>647384
> полые планеты
Заброшенная Сфера Дайсона предтеч, обжитая одичалыми потомками.
> восьмерочные орбиты
Видел в андроид-игре, где надо было планеты вокруг чёрных дыр запускать.
> звук в космосе
Элемент H.U.D. оператора боевого орудия космолёта. Звуки выстрелов генерируются системой ведения огня и передаются ему на слуховой нерв, для интуитивного ощущения боя, вместе с визуальными данными на глазной нерв.
Аноним 16/04/21 Птн 09:09:39 647505577
>>647503
>Видел в андроид-игре, где надо было планеты вокруг чёрных дыр запускать.
Меня терзают смутные сомнения что н-тела они не реализовали. Просто так в игрушке я тоже тебе хоть синусоидальную траекторию сделаю. ИРЛ восьмерочная орбита это как сидение на двух стульях одновременно. Когда эти стулья во многих а.е. друг от друга. Только твоя мамаша может на них усидеть, но тогда это не она будет вокруг звезд вращаться, а они вокруг неё.

>Элемент H.U.D. оператора боевого орудия космолёта. Звуки выстрелов генерируются системой ведения огня и передаются ему на слуховой нерв, для интуитивного ощущения боя, вместе с визуальными данными на глазной нерв.
А это норм, неплохо.
Аноним 16/04/21 Птн 09:14:33 647507578
1618553673058.png 45Кб, 350x241
350x241
>>647505
> Меня терзают смутные сомнения
Ну да, признаю, игра не может быть научным пруфом для ТНФ. Но вот смотри, я тут погуглил и загуглились некие лемнискатные орбиты. Но это чистый матан, не наблюдаемый в природе (как я понел).
Аноним 16/04/21 Птн 11:12:46 647519579
>>647507
>Запускаешь планету в тесной двойной системе
>Планета проходит через полость Роша
>Какой еще нахуй приливный разрыв?
Аноним 16/04/21 Птн 11:17:43 647520580
Обьясните доступно, что такое гравитационный маневр. И каким образом планета или еще какой обьект отдают свою энергию нашему космолету.
Максимум каких скоростей можно добиться при таких раскладах?
Если этот маневра сделать у какой-нить черной дыры, то экипаж распидарасит неебическим ускорением?
Аноним 16/04/21 Птн 11:40:30 647521581
>>647520
ну вот есть разность скоростей тела и планеты вокруг которой маневрировать

и вот эту дельта вэ можно забрать у тела и добавить в около произвольном направлении, при этом относительно плонетки скорость тела на входе и выходе будет одна и та же
алсо можно ещё газануть в перицентре для дополнительного ускорения, манёвр оберта

максимум для каждого случая и планеты отдельный, доступно не получится
у черной дыры экипаж распидорасит не гравитационный манёвр, там квази сила, просто крученое пространство
Аноним 16/04/21 Птн 11:49:03 647522582
>>647521

Ну в Интерстелларе они же выполнили гравитационный маневр у ЧД и их не распидарасило.
Насколько я помню, они пожертвовали временем, но маневр выполнили.
Учитывая, что консультантом был Кип Торн, а не Виктория Боня - получилось все по науке, хоть и с допущениями, если я правильно все понял.
Аноним 16/04/21 Птн 11:53:04 647523583
>>647522
>Ну в Интерстелларе они же выполнили гравитационный маневр у ЧД и их не распидарасило.
Не говорите ему что интерстеллар не документальный фильм
Аноним 16/04/21 Птн 12:02:28 647524584
>>647523
В Интерстелларе сверхмассивная ЧД, ты даже не заметишь, как горизонт событий пересечешь. Так что в этом плане, там все нормально показано (за исключением размеров ЧД, которая должна быть существенно больше).
Аноним 16/04/21 Птн 12:09:57 647525585
>>647520
Короче это обмен угловым моментами при сохранение импульса или наоборот, обмен импульсами при сохранение углового момента.
Скорость зависит от орбитальных характеристик тел. При удачном выборе легко добиваться третьей космической.
Орбитальное движение является инерциальным. Принцип эквивалентности все дела. Даже если рядом с ЧД будут двигаться, то не почувствуют перегрузку. Но у ЧД есть другие бяки вроде буквально разрывающего жопу прилива и аккреционного диска движущегося с релятивистской скоростью.
Аноним 16/04/21 Птн 12:12:40 647527586
>>647524
> сверхмассивная ЧД, ты даже не заметишь, как горизонт событий пересечешь.
анонимные академики итт же обоссали этот высер научпопа, горизонт может пересечь без распидораса только элементарная частица
Аноним 16/04/21 Птн 12:13:21 647528587
>>647524
Там ЧД не превышает 10 солнечных масс. Ибо там близкая планета летает и нет мазеров из газа поблизости.
Ну перед подходом к горизонту есть и другие эффекты вроде фотонной сферы и раскручиванию вокруг своей оси.
Аноним 16/04/21 Птн 12:32:26 647530588
>>647528

На столь малом расстоянии приливная гравитация черной дыры (см. главу 4) особенно сильна. Она растягивает планету Миллер в направлениях к Гаргантюа и от нее и сжимает «по бокам» (рис. 6.1).

Сила этих растяжений и сжатий обратно пропорциональна квадрату массы Гаргантюа. Почему это так? Чем больше масса Гаргантюа, тем больше ее окружность, а значит, тем меньше разница между гравитационными силами, действующими на разные части планеты, то есть тем слабее приливные силы (см. ньютоновскую интерпретацию приливных сил, рис. 4.8). Исходя из этого можно сделать вывод, что масса Гаргантюа превышает солнечную не менее чем в 100 миллионов раз.

Будь Гаргантюа не такой массивной, планету Миллер разорвало бы на части! Во всех дальнейших рассуждениях я буду считать, что масса Гаргантюа равна массе 100 миллионов Солнц. Например, объясняя в главе 17, как приливные силы Гаргантюа могут вызвать на планете Миллер гигантские волны, которые обрушиваются на «Рейнджер», я исхожу из этого значения.

Длина окружности горизонта событий черной дыры пропорциональна ее массе. Для Гаргантюа, масса которой составляет 100 миллионов Солнц, окружность горизонта приблизительно совпадает по размерам с орбитой движения Земли вокруг Солнца — около миллиарда километров. Неслабо!

Посовещавшись со мной, команда по созданию визуальных эффектов Пола Франклина использовала именно это значение. Физики принимают радиус черной дыры равным длине ее окружности, деленной на 2π (около 6,28). Из-за чудовищных искривлений пространства внутри черной дыры это значение не соответствует ее истинному радиусу, оно не равно расстоянию от горизонта до центра дыры, если измерять его в нашей Вселенной. Зато оно равно радиусу (половине диаметра) горизонта событий, если измерять его из балка (см. рис. 6.3). Понимаемый так радиус Гаргантюа составляет примерно 150 миллионов километров; столько же, сколько радиус орбиты Земли вокруг Солнца.
Аноним 16/04/21 Птн 12:39:43 647532589
>>647524
>ты даже не заметишь, как горизонт событий пересечешь
Взаимодействие не может передаваться из-за горизонта событий, тебя распидорасит на элементарные частицы
Аноним 16/04/21 Птн 12:40:44 647533590
>>647530
Все равно не теняет на сверхмассивную. Такие в шаровых скоплениях всем кластером выкидывали.
Аноним 16/04/21 Птн 12:44:52 647534591
>>647532

После того, как ты пересечешь горизонт событий и начнешь приближаться к сингулярности - расипидарасит, разумеется.
Но сам горизонт ты пересечешь живым, в случае со сверхмассивной ЧД.
У ЧД звездной массы, тебя спагеттизирует еще при подлете к горизонту.
Аноним 16/04/21 Птн 12:47:13 647535592
>>647533

Тянет, тянет.
Стрелец А, вокруг которого крутится наша галактика имеет массу в 4 миллиона солнечных.
Аноним 16/04/21 Птн 13:01:53 647538593
>>647534
А как же быть с раскручиванием и "стеной огня"?
У сверхмассивной ЧД на последней устойчивой орбите накапливается столько вещества, что на большую звезду хватит.
В добавок есть эргосфера, которая не только тебя раскручивает, но еще может откинуть раскрученного подальше с релятивистской скоростью.
Аноним 16/04/21 Птн 13:09:39 647539594
>>647538

Почитай Интерстеллар - Наука за кадром от Кипа Торна.
Он там все очень подробно разьясняет.
Серьезно, охуенная книжка.
Он сам делит события в фильме на 3 категории.

Научные истины, обоснованные предположения и домыслы.

За акрецционный диск он в ней тоже поясняет.
Аноним 16/04/21 Птн 13:09:57 647540595
>>647534
Каким образом ты собрался живым пересекать горизонт событий если как только первый слой атомов за него пройдет он оторвется от тебя?
Аноним 16/04/21 Птн 13:16:42 647542596
>>647538
Аккреционный диск Гаргантюа и отсутствие джета
Типичный аккреционный диск и его джет испускают рентгеновское излучение, гамма-лучи, радиоволны и свет; и мощь этого излучения такова, что оно уничтожило бы всех людей, находящихся неподалеку. Чтобы избежать этого, Кристофер Нолан и Пол Франклин снабдили Гаргантюа чрезвычайно слабым диском.

Ну как слабым… Слабым по стандартам типичных квазаров. Вместо температуры в сотню миллионов градусов, как у диска типичного квазара, температура диска Гаргантюа – «всего» несколько тысяч градусов (как на поверхности Солнца). Поэтому диск Гаргантюа испускает много света, но почти не испускает рентгеновских и гамма-лучей. Когда газ настолько «прохладный», тепловое движение атомов слишком медленное, чтобы диск был толстым. В итоге он «тонко размазан» по экваториальной плоскости Гаргантюа.

Такой диск может быть у «проголодавшейся» черной дыры, то есть дыры, которая за последние миллионы или более лет не растерзала ни одной звезды. В этом случае магнитное поле, изначально привязанное к плазме диска, истощится, а джет, который оно подпитывало, – исчезнуть. Таков диск Гаргантюа: тонкий, без джета и относительно безопасный для людей. Относительно.

Диск Гаргантюа заметно отличается от изображений тонких дисков из трудов астрофизиков, поскольку в их иллюстрациях отсутствует одна важная особенность – гравитационное линзирование диска черной дырой. В «Интерстеллар» линзирование есть, поскольку Крис настаивал на зрительной достоверности.

Перед Эжени фон Танзелманн стояла задача прогнать аккреционный диск через компьютерную программу гравитационного линзирования, о которой я писал в главе 8. Первым делом, чтобы оценить результат линзирования, Эжени использовала бесконечно тонкий диск, лежащий точно в экваториальной плоскости Гаргантюа. Для этой книги она предоставила более наглядный вариант такого диска, состоящий из равномерно распределенных цветных участков.
Аноним 16/04/21 Птн 13:42:10 647546597
>>647542
Ну вот уже допущение, что диск должен быть очень тонким.
По идеи же диск сжимает собственное магнитное поле, и при его ослаблении диск разжимается и превращается в тор.
И это не отменяет того, что на последней устойчивой орбите накапливается много вещества, которому для падения трудно сбросить свой угловой момент. А у холодных дисков из-за низкой светимости там будет уже считай очень плотная атмосфера.
И изучение из фотонное сферы некуда не надеется само по себе.
Аноним 16/04/21 Птн 19:11:39 647597598
>>647520
Представь, что ты бросаешь мячик в лобовое стекло движущегося грузовика. С точки зрения грузовика мячик отскочит от стекла с той же скоростью, с которой прилетел, а с точки зрения неподвижного наблюдателя отскок от движущегося объекта придаст мячику дополнительную скорость.

Это и есть практически полная аналогия гравитационного маневра. Аппарат (или мячик) совершает оборот (или отскок) об движущийся объект и крадёт часть его импульса.

>Максимум каких скоростей можно добиться при таких раскладах?
Если принять за скорость аппарата vапп, а за скорость планеты vплан, то максимальная скорость после маневра — vапп+2×vплан.

Это легко видеть из того, что с точки зрения планеты аппарат сначала сближается с ней со скоростью vапп+vплан, а потом удаляется со скоростью vапп+vплан. С точки зрения неподвижного наблюдателя, соответственно, аппарат сначала движется к планете со скоростью vапп, а потом удаляется от неё со скоростью vапп+2×vплан.

Но это идеальный случай, возможный только для случая, когда направление аппарата после маневра меняется на 180°. Обычно там и планета не настолько массивная, чтобы так круто изменить траекторию аппарата, и сама траектория проходит не под нужным углом.

>Если этот маневра сделать у какой-нить черной дыры, то экипаж распидарасит неебическим ускорением?
Смотря насколько близко к ЧД пролетать, и насколько она массивная. Если близко, и к крупной — то да, там всех людей размажет кашицей по потолку.
Аноним 16/04/21 Птн 19:35:43 647601599
videoplayback.webm 40619Кб, 1280x720, 00:01:54
1280x720
Аноним 16/04/21 Птн 21:28:27 647638600
>>647597

То есть, имеет значение только скорость обьекта, а не масса?
Аноним 16/04/21 Птн 21:58:05 647647601
>>647638
от массы зависит кривизна пространства и максималка разворота плюс ещё важен радиус, в атмосфере с литосферой этот манёвр не такой эффектный
но если разницы скоростей мало то мало и смысла вне зависимости от массы
Аноним 17/04/21 Суб 09:13:32 647743602
>>647434
>Почему мы считаем, что вселенная расширяется?
Ебать, потому что об этом свидетельствуют все данные по этому вопросу, которые у нас сегодня есть.
>На основании каких наблюдений
https://ru.m.wikipedia.org/wiki/Закон_Хаббла
https://elementy.ru/nauchno-populyarnaya_biblioteka/434741/Est_li_problemy_s_soglasovaniem_skorosti_rasshireniya_Vselennoy
https://ru.m.wikipedia.org/wiki/Ускоряющаяся_Вселенная
https://elementy.ru/nauchno-populyarnaya_biblioteka/435153/O_temnoy_energii_zamolvite_slovo
>и в какое время
Начиная с Эддвина Хаббла, т.е. с 1920-х годов, и заканчивая сегодняшним временем.
>и в каком кластере космоса были сделаны измерения?
В размере видимой части Вселенной.
>Может, это не вселенная расширяется все, а лишь один какой-то кластер её, в котором мы и наблюдали расширение, экстраполировав его на всю вселенную?
>Попробуем просуммировать сказанное. Начнем с того, что нового мы знаем о нашей Вселенной в свете теории космологической инфляции.
>Ее размер, скорее всего, конечен, но несомненно огромен. Формально для него мы можем лишь дать нижний предел: в сто раз больше, чем расстояние до горизонта. Мы знаем это из современных измерений кривизны Вселенной. Но в сто раз больше — это крайне маловероятно; скорее, в миллиарды или, скажем, на 20 или на 50 порядков больше. Мы понимаем это из характера космологической инфляции — это экспоненциальный процесс. Ее продолжительность неизвестна и до какой-то степени случайна, а продолжительность стоит в показателе степени. Если для создания Вселенной размером с ее наблюдаемую часть инфляция должна была продолжаться по меньшей мере 10-35 с, то при времени раздувания 210-35 с ее размер будет на 30 порядков больше наблюдаемого (цифры приблизительны). То есть мы с подавляющей вероятностью видим лишь «микроскопическую» часть Вселенной.
>Экспоненциальность инфляции говорит и о том, что за горизонтом — то же самое, что мы видим: такая же крупномасштабная структура, такое же соотношение между обычной и темной матерями и темной энергией. По крайней мере, наш пейзаж скорее всего продолжается на расстояниях несравненно больших, чем размер горизонта. 
https://coollib.com/b/428217-boris-evgenevich-shtern-proryiv-za-kray-mira
Аноним 17/04/21 Суб 13:42:46 647788603
Почему на Луну не отправляли животных?
Аноним 17/04/21 Суб 13:53:03 647789604
>>647788
отправляли, раз пять
Аноним 17/04/21 Суб 14:19:26 647794605
Аноним 17/04/21 Суб 15:26:58 647808606
>>647794
>А зачем?
Умаслить Зверя Луны после зафейленного америкой ритуала порабощения Луны в угоду властям США, чтобы Зверь свой гнев не на всех людей низверг, а только на непосредственно ответственных.
Инфа 146%
Аноним 17/04/21 Суб 17:47:03 647823607
Удешевление космических полетов это инженерная задача или нужно ждать прорывов в физике?
Аноним 17/04/21 Суб 18:15:38 647828608
>>647823
Политическая в первую очередь. Потом инженерная.
Аноним 17/04/21 Суб 18:18:49 647829609
>>647828
Если Байден даст добро и Маск будет работать ещё усерднее я смогу купить путевку на орбиту?
Аноним 17/04/21 Суб 18:22:24 647830610
>>647823
>>647829
вопрос очень общий, куда и за какой прайс
Аноним 17/04/21 Суб 18:47:34 647831611
>>647830
На экскурсию, хотя бы на орбиту, по цене поездки на Карибы, например.
Аноним 17/04/21 Суб 21:05:57 647842612
>>647829
Если бидон решит потратить несколько сотен миллиардов на пусковую петлю, а маска повесят, то хоть каждый день летай.
Аноним 17/04/21 Суб 21:18:34 647843613
>>647842
у бидона плохо с айсикью, только в обычную петельку сможет
Аноним 17/04/21 Суб 21:43:20 647849614
>>647843
Будем надеяться что случайно пару ноликов бюджету насы припишет
Аноним 17/04/21 Суб 21:53:47 647851615
>>647842
Это будет экономически оправданный проект или давайпососательный?
Аноним 17/04/21 Суб 21:57:27 647853616
Аноним 18/04/21 Вск 08:11:23 647882617
>>647843
Да он бы и в лошадиную.
Аноним 18/04/21 Вск 08:58:57 647883618
>>647849
Ему скажут, что космонавтика это один из символов угнетения белыми всех остальных и что эти деньги могли бы пойти на пенсии для негров, и он подпишет указ, чтобы всех их разогнали, а активы муска национализировали и продали китайцам.
Аноним 18/04/21 Вск 12:09:00 647898619
154320797615563[...].jpg 30Кб, 400x376
400x376
>>647883
Нет, пожалуйста, не надо.
Аноним 18/04/21 Вск 14:06:17 647906620
AcroRd32ykexHij[...].png 61Кб, 809x549
809x549
На мощности в 480Кв насколько быстро можно будет долететь до марса в окне 2030-х годов?
Аноним 18/04/21 Вск 14:12:21 647907621
>>647906
Зависит от корабля и движителя.
Аноним 18/04/21 Вск 23:41:47 647987622
1-giuseppe-piaz[...].jpg 304Кб, 743x900
743x900
Аноним 19/04/21 Пнд 08:53:09 648045623
>>647535
> Стрелец А, вокруг которого крутится наша галактика
Нет.
Галактика намного тяжелее стрельца. И крутится она вокруг своей скрытой массы, которую мочёные всё никак не задефайнят.
Аноним 10/05/21 Пнд 23:24:32 654525624
если всё человечество объединит свои силы, ресурсы, технологии и прочее сможем вообще когда-нибудь построить сферу дайсона? или это в принципе не возможно?
Настройки X
Ответить в тред X
15000
Макс объем: 40Mб, макс кол-во файлов: 4
Кликни/брось файл/ctrl-v
Стикеры X
Избранное / Топ тредов